NURS 6512 Week 8: Assessment of the Musculoskeletal System

NURS 6512 Week 8: Assessment of the Musculoskeletal System

The purpose of this discussion is to evaluate the following case study:

A 42-year-old male reports pain in his lower back for the past month. The pain sometimes radiates to his left leg. In determining the cause of the back pain, based on your knowledge of anatomy, what nerve roots might be involved? How would you test for each of them? What other symptoms need to be explored? What are your differential diagnoses for acute low back pain? Consider the possible origins using the Agency for Healthcare Research and Quality (AHRQ) guidelines as a framework. What physical examination will you perform? What special maneuvers will you perform?

For this patient complaining of lower back pain, there are many nerves that may be involved. The lumbar plexus, femoral nerve, sacral nerve, sciatic nerve, posterior cutaneous nerve, and pudendal nerve are all potential causes of lower back pain (Ball et al., 2023). Patients with Lumbosacral radiculopathy ( herniated lumbar discs) can be assessed by physical examination techniques. Nerve root L4 can be tested by asking the patient to squat and rise back up. A positive result will cause the patient to have pain and weakness on the affected side. Knee jerk reflex should also be tested, as the affected side would have a diminished response. Nerve root L5 is tested by placing pressure on the first toe of each foot. The patient will attempt to pull pressure against the examiner. A positive result will cause the patient to give way to the affected side. The patient will also be asked to demonstrate heel walking, which is difficult for patients with L4 or L5 herniated discs. Patients with S1 herniations have difficulty with plantar flexion of the great toe, walking on their toes, and diminished ankle reflexes (Ball et al., 2023). 

This patient should receive a full evaluation and assessment before any conclusions are reached. Adults presenting with low back pain should always be assessed for abdominal aortic aneurysm. Some patients present with unusual symptoms, and back pain should never be ignored. Caucasian males are most likely to have the disease, and being over the age of 60 and a current or previous smoker places the patient at increased risk (Ullery et al., 2018).

Online Nursing Essays

Struggling to Meet Your Deadline?

Get your assignment on NURS 6512 Week 8: Assessment of the Musculoskeletal System done on time by medical experts. Don’t wait – ORDER NOW!

Patients should be asked to give a thorough health history. Specifically, the patient should be asked if he has ever experienced pain and discomfort like this before. The examiner should determine what the patient was doing when the symptoms began, the quality of the pain, the exact location of the pain, and if anything relieves or worsens the pain. These factors will help guide the treatment plan for this patient. Palpation of the abdomen will be essential for the assessment to determine if the patient has tenderness or guarding that would lead the diagnosis away from neurological or musculoskeletal and toward organ disease. Patients with kidney infections or renal stones also often present with flank pain on one side. The kidneys should be palpated for tenderness.

Conditions that should be considered for this patient include:

1) Herniated Disc

2) Sciatica

3) Strained Muscle

4) Osteoarthritis

5) Renal Stones

References

Ball, J. W., Dains, J. E., Flynn, J. A., Solomon, B. S., & Stewart, R. W. (2023). Seidel’s guide to physical examination: An interprofessional approach (10th ed.). Elsevier Mosby.

Ullery, B. W., Hallett, R. L., & Fleischmann, D. (2018). Epidemiology and contemporary management of abdominal aortic aneurysms. Abdominal Radiology, 43(5), 1032–1043. https://doi.org/10.1007/s00261-017-1450-7

Zibis, A. H., Mitrousias, V. D., Klontzas, M. E., Karachalios, T., Varitimidis, S. E., Karantanas, A. H., & Arvanitis, D. L. (2018). Great trochanter bursitis vs sciatica, a diagnostic–anatomic trap: Differential diagnosis and brief review of the literature. European Spine Journal, 27(7), 1509–1516. https://doi.org/10.1007/s00586-018-5486-3

NURS 6512 Week 8: Assessment of the Musculoskeletal System

NURS 6512 Week 8: Assessment of the Musculoskeletal System

Week 8: Assessment of the Musculoskeletal System

Patient Information:

JD, 15, M, Caucasian

S.

CC: JD complains of dull pain in both knees

HPI: JD is a 15-year-old Caucasian male presents with dull pain in both knees bilaterally and occasional catching/clicking with ambulation. JD reports the pain started 3 weeks ago after he collided with a runner at home plate during a baseball game. JD reports the pain as dull and intermittent on a scale of 5/10. He has not taken anything for the pain at this point. Walking and physical activity make the pain worse and ice/elevation and resting the legs make the pain better. JD reports he has not been able to participate in his baseball games since the injury occurred.

Current Medications: no current medications

Allergies: NKDA, no environmental allergies, no food allergies

PMHx: No previous medical history or surgeries

Soc Hx: Denies the use of cigarettes, denies etoh use, denies use of marijuana and illicit drugs. Freshman in high school, drum major in the band and plays basketball and baseball.

Fam Hx: Mother is 38 y/o with hypertension, Father is 35 with diabetes, Maternal grandmother is living with hypertension and hyperlipidemia, Maternal grandfather is living with hypertension and BPH, Paternal grandmother is living with diabetes and hypertension, Paternal grandfather is living with hyperlipidemia, Sister is 7 years old with no medical issues.

ROS:

Example of Complete ROS:

GENERAL:  Denies weight loss, fever, chills, weakness or fatigue.

HEENT:  Eyes: Denies visual loss, blurred vision, double vision, or yellow sclerae. Ears, Nose, Throat . Denies hearing loss, sneezing, congestion, runny nose or sore throat.

SKIN:  Denies rash or itching.

CARDIOVASCULAR:  Denies chest pain, chest pressure, or chest discomfort. No palpitations or edema.

RESPIRATORY:  Denies shortness of breath, cough, or sputum.

GASTROINTESTINAL:  Denies anorexia, nausea, vomiting, or diarrhea. No abdominal pain or blood.

GENITOURINARY:  No burning on urination and denies urgency.

NEUROLOGICAL:  Denies headache, dizziness, syncope, paralysis, ataxia, numbness or tingling in the extremities. No change in bowel or bladder control.

MUSCULOSKELETAL:  Reports clicking/catching sensation under bilateral knee caps during ambulation

HEMATOLOGIC:  Denies anemia, bleeding or bruising.

LYMPHATICS:  Denies enlarged nodes. No history of splenectomy.

PSYCHIATRIC:  Denies history of depression or anxiety.

ENDOCRINOLOGIC:  Denies reports of sweating, cold or heat intolerance. No polyuria or polydipsia.

ALLERGIES:  Denies history of asthma, hives, eczema or rhinitis.

Click here to ORDER an A++ paper from our Verified MASTERS and DOCTORATE WRITERS: NURS 6512 Week 8: Assessment of the Musculoskeletal System

O.

General: JD presents alert and oriented, appropriate hygiene, and without gait disturbances

Cardio: S1/S2, no murmurs or adventitious heart sounds heard

Resp: Lung sounds clear throughout lobes bases bilaterally, respirations even and unlabored

Musculoskeletal: Popping sounds heard with flexion of knees bilaterally, ballottement positive bilaterally.

Diagnostic results: Xray of bilateral knees, MRI of knees bilaterally

A.

Differential Diagnoses:

Bursitis: Bursitis is common in individuals who are often in the kneeling or squatting position. The prolonged kneeling causes inflammation of the bursa sac. This patient is a catcher on his baseball team and is squatting for extended periods of time (LeManac et al., 2012). Bursitis presents as pain and swelling in the knee joints exacerbated by the kneeling position.

ACL tear: Due to this patient being active in sports he could have torn his ACL. An ACL tear can present with symptoms such as popping, pain, swelling, and discomfort while walking. ACL tears can happen with a sudden change in directions, quickly slowing down, jumping, and landing wrong (Orthoinfo, 2014).

Patellar Tendonitis: This is reported as pain at the tendon where the patella connects to the tibia. This is common in individuals who participate in jumping activities such as basketball and volleyball players. Also known as “jumper’s knee”. This is caused by tiny tears to the patella tendon from increased stress over time causing weakness and pain to the affected area (Mayo Clinic, 2019). This patient plays basketball in high school and is very active and has high probability of this injury

Patellar fracture: This injury can take place due to blunt force to the affected area. The patient could have fallen on the basketball court or been hit by another player during a baseball game causing trauma to the knee. This presents with the inability to walk or straighten the leg. Severe pain or swelling and oftentimes bruising to the area (Boston Medical Center, n.d.).

Meniscus tear: The meniscus is cartilage that absorbs the impact of fast movements and the shock of body movements. The cartilage can tear with sudden movements or changing of directions. This patient participates in sports that could cause injuries such as this especially basketball. Meniscus injuries present with pain, swelling, locking/catching at the knee joint (John Hopkins Medicine, n.d.).

References

Anterior cruciate Ligament (acl) injuries – OrthoInfo – AAOS. OrthoInfo. (2014). https://orthoinfo.aaos.org/en/diseases–conditions/anterior-cruciate-ligament-acl-injuries/.

Le Manac, h, A. P., Ha, C., Descatha, A., Imbernon, E., & Roquelaure, Y. (2012). Prevalence of knee bursitis in the workforce. Occupational Medicine62(8), 658–660. https://doi-org.ezp.waldenulibrary.org/10.1093/occmed/kqs113

Mayo Foundation for Medical Education and Research. (2019, October 16). Patellar tendinitis. Mayo Clinic. https://www.mayoclinic.org/diseases-conditions/patellar-tendinitis/symptoms-causes/syc-20376113.

Patellar (Kneecap) Fracture. Boston Medical Center. (n.d.). https://www.bmc.org/patient-care/conditions-we-treat/db/patellar-kneecap-fracture.

Torn meniscus. Johns Hopkins Medicine. (n.d.). https://www.hopkinsmedicine.org/health/conditions-and-diseases/torn-meniscus.

Week 8: Assessment of the Musculoskeletal System

A 46-year-old man walks into a doctor’s office complaining of tripping over doorways more frequently. He does not know why. What could be the causes of this condition?

Without the ability to use the complex structure and range of movement afforded by the musculoskeletal system, many of the physical activities individuals enjoy would be curtailed. Maintaining the health of the musculoskeletal system will ensure that patients live a life of full mobility. One of the most basic steps that can be taken to preserve the health of the musculoskeletal system is to perform an assessment.

This week, you will explore how to assess the musculoskeletal system.

Learning Objectives

Students will:

  • Evaluate abnormal musculoskeletal findings
  • Apply concepts, theories, and principles relating to health assessment techniques and diagnoses for the musculoskeletal system

Photo Credit: SCIEPRO/Science Photo Library/Getty Images

 

Learning Resources-Week 8: Assessment of the Musculoskeletal System

Note: To access this week’s required library resources, please click on the link to the Course Readings List, found in the Course Materials section of your Syllabus.

Required Readings

Ball, J. W., Dains, J. E., Flynn, J. A., Solomon, B. S., & Stewart, R. W. (2015). Seidel’s guide to physical examination (8th ed.). St. Louis, MO: Elsevier Mosby.

    • Review of Chapter 4, “Vital Signs and Pain Assessment” (pp. 50-63)
  • Chapter 21, “Musculoskeletal System” (pp. 501-543)This chapter describes the process of assessing the musculoskeletal system. In addition, the authors explore the anatomy and physiology of the musculoskeletal system.

Dains, J. E., Baumann, L. C., & Scheibel, P. (2016). Advanced health assessment and clinical diagnosis in primary care (5th ed.). St. Louis, MO: Elsevier Mosby.

    • Chapter 22, “Limb Pain” (pp. 356-374)This chapter outlines how to take a focused history and perform a physical exam to determine the cause of limb pain. It includes a discussion of the most common tests used to assess musculoskeletal disorders.
  • Chapter 24, “Low Back Pain (Acute)” (pp. 288-300)The focus of this chapter is the identification of the causes of lower back pain. It includes suggested physical exams and potential diagnoses.

Sullivan, D. D. (2019). Guide to clinical documentation (3rd ed.). Philadelphia, PA: F. A. Davis.

    • Chapter 2, “The Comprehensive History and Physical Exam” (“Muscle Strength Grading”; p. 29)
  • Chapter 4, “Pediatric Preventative Care Visits” (“Documentation of Important Components of Age Specific Physical Exams and Sports Pediatric Sports Participation Physical Exam”; pp. 106-107)

Note: Download this Adult Examination Checklist and Physical Exam Summary: Abdomen to use during your practice musculoskeletal examination.

Seidel, H. M., Ball, J. W., Dains, J. E., Flynn, J. A., Solomon, B. S., & Stewart, R. W. (2011). Adult examination checklist: Guide for musculoskeletal assessment. In Mosby’s guide to physical examination (7th ed.). St. Louis, MO: Elsevier Mosby. This Adult Examination Checklist: Guide for Musculoskeletal Assessment was published as a companion to Seidel’s guide to physical examination (8th ed.), by Ball, J. W., Dains, J. E., & Flynn, J. A. Copyright Elsevier (2015). From https://evolve.elsevier.com/

Seidel, H. M., Ball, J. W., Dains, J. E., Flynn, J. A., Solomon, B. S., & Stewart, R. W. (2011). Physical exam summary: Musculoskeletal system. In Mosby’s guide to physical examination (7th ed.). St. Louis, MO: Elsevier Mosby. This Musculoskeletal System Physical Exam Summary was published as a companion to Seidel’s guide to physical examination (8th ed.), by Ball, J. W., Dains, J. E., & Flynn, J. A. Copyright Elsevier (2015). From https://evolve.elsevier.com/

Katz, J. N., Lyons, N., Wolff, L. S., Silverman, J., Emrani, P., Holt, H. L., & …Losina, E. (2011). Medical decision-making among Hispanics and non-Hispanic Whites with chronic back and knee pain: A qualitative study. BMC Musculoskeletal Disorders, 12(1), 78–85. Retrieved from the Walden Library databases. This study examines the medical decision making among Hispanics and non-Hispanic whites. The authors also analyze the preferred information sources used for making decisions in these populations.

University of Virginia. (n.d.). Introduction to radiology: An online interactive tutorial. Retrieved from http://www.med-ed.virginia.edu/courses/rad/index.html. This website provides an introduction to radiology and imaging. For this week, focus on skeletal trauma in musculoskeletal radiology.

Smuck, M., Kao, M., Brar, N., Martinez-Ith, A., Choi, J., & Tomkins-Lane, C. C. (2014). Does physical activity influence the relationship between low back pain and obesity? The Spine Journal, 14(2), 209–216. doi:10.1016/j.spinee.2013.11.010 Retrieved from the Walden Library Databases.

Shiri, R., Solovieva, S., Husgafvel-Pursiainen, K., Telama, R., Yang, X., Viikari, J., Raitakari, O. T., & Viikari-Juntura, E. (2013). The role of obesity and physical activity in non-specific and radiating low back pain: The Young Finns study. Seminars in Arthritis & Rheumatism, 42(6), 640–650. doi:10.1016/j.semarthrit.2012.09.002. Retrieved from the Walden Library Databases.

Required Media – Week 8: Assessment of the Musculoskeletal System

Online media for Seidel’s Guide to Physical Examination

In addition to this week’s resources, it is highly recommended that you access and view the resources included with the course text, Seidel’s Guide to Physical Examination. Focus on the videos and animations in Chapter 21 that relate to the assessment of the musculoskeletal system. Refer to the Week 4 Learning Resources area for access instructions on https://evolve.elsevier.com/.

Optional Resources FOR Week 8: Assessment of the Musculoskeletal System

LeBlond, R. F., Brown, D. D., & DeGowin, R. L. (2014). DeGowin’s diagnostic examination (10th ed.). New York, NY: McGraw Hill Medical.

  • Chapter 13, “The Spine, Pelvis, and Extremities” (pp. 585–682)In this chapter, the authors explain the physiology of the spine, pelvis, and extremities. The chapter also describes how to examine the spine, pelvis, and extremities.
 

The body is constantly sending signals about its health. One of the most easily recognized signals is pain. Musculoskeletal conditions comprise one of the leading causes of severe long-term pain in patients. The musculoskeletal system is an elaborate system of interconnected levers that provide the body with support and mobility. Because of the interconnectedness of the musculoskeletal system, identifying the causes of pain can be challenging. Accurately interpreting the cause of musculoskeletal pain requires an assessment process informed by patient history and physical exams.

In this Discussion, you will consider case studies that describe abnormal findings in patients seen in a clinical setting.

Note: By Day 1 of this week, your Instructor will have assigned you to one of the following specific case studies for this Discussion. Also, your Discussion post should be in the Episodic/Focused SOAP Note format, rather than the traditional narrative style Discussion posting format. Refer to Chapter 2 of the Sullivan text and the Episodic/Focused SOAP Template in the Week 5 Learning Resources for guidance. Remember that all Episodic/Focused SOAP notes have specific data included in every patient case.

Case 1: Back Pain

A 42-year-old male reports pain in his lower back for the past month. The pain sometimes radiates to his left leg. In determining the cause of the back pain, based on your knowledge of anatomy, what nerve roots might be involved? How would you test for each of them? What other symptoms need to be explored? What are your differential diagnoses for acute low back pain? Consider the possible origins using the Agency for Healthcare Research and Quality (AHRQ) guidelines as a framework. What physical examination will you perform? What special maneuvers will you perform?

Case 2: Ankle Pain

A 46-year-old female reports pain in both of her ankles, but she is more concerned about her right ankle. She was playing soccer over the weekend and heard a “pop.” She is able to bear weight, but it is uncomfortable. In determining the cause of the ankle pain, based on your knowledge of anatomy, what foot structures are likely involved? What other symptoms need to be explored? What are your differential diagnoses for ankle pain? What physical examination will you perform? What special maneuvers will you perform? Should you apply the Ottowa ankle rules to determine if you need additional testing?

Case 3: Knee Pain

A 15-year-old male reports dull pain in both knees. Sometimes one or both knees click, and the patient describes a catching sensation under the patella. In determining the causes of the knee pain, what additional history do you need? What categories can you use to differentiate knee pain? What are your specific differential diagnoses for knee pain? What physical examination will you perform? What anatomic structures are you assessing as part of the physical examination? What special maneuvers will you perform?

To prepare:

With regard to the case study you were assigned:

  • Review this week’s Learning Resources, and consider the insights they provide about the case study.
  • Consider what history would be necessary to collect from the patient in the case study you were assigned.
  • Consider what physical exams and diagnostic tests would be appropriate to gather more information about the patient’s condition. How would the results be used to make a diagnosis?
  • Identify at least five possible conditions that may be considered in a differential diagnosis for the patient.

Note: Before you submit your initial post, replace the subject line (“Discussion – Week 8”) with “Review of Case Study ___.” Fill in the blank with the number of the case study you were assigned.

By Day 3

Post an episodic/focused note about the patient in the case study to which you were assigned using the episodic/focused note template provided in week 5 resources. Provide evidence from the literature to support diagnostic tests that would be appropriate for each case. List five different possible conditions for the patient’s differential diagnosis and justify why you selected each.

Patient Information:

K.M. – 46-yearld-old Caucasian female

CASE 2 Ankle Sprain

S.

CC (chief complaint) Bilateral ankle pain with emphasis to right ankle

HPI:

The patient is a 46-year-old causation female presenting with bilateral ankle pain.  She states hearing “a pop” in her right ankle while playing soccer over the weekend (5 days ago), so she is seeking treatment/diagnosis.  The patient felt like her foot became caught in the grass, which caused her to pitch forward; the “pop” sound felt attributed to the top or outside of her ankle.  Current pain is to bilateral ankles with a pain rating of 2 to the left ankle and 6 to the right ankle at the time of injury.  Pain in the right ankle was immediate, and swelling began to occur several hours later.  Her pain is described as a constant throbbing and an overall feeling of instability. 

Pain is worsened when weight is applied during ambulation.  Ice packs were placed on the ankle intermittently to reduce swelling/inflammation, ankelwas also elevated to reduce swelling, and Motrin 400mg PO every 6 hours was taken for pain/inflammation.  These interventions helped to subside swelling and pain mildly but were deemed ineffective.  Pain levels are elevated when bearing weight and attempting to walk.  Two days post-status, the ankle began to show signs of bruising with increased swelling.

Current Medications:

Ibuprofen 400mg PO tablet – taken every 6 hours for pain/swelling (last dose 7am this morning)

Bone Health Supplement PO capsule (Vitamins C, D, K, and magnesium) – 1 capsule daily for (6 years) for bone strengthening and prevention of osteoporosis (last dose this morning).

Allergies:

  1. Medications – denies
  2. Food – denies
  3. Latex – denies
  4. Environmental – denies

PMHx: Denies medical history

Soc Hx:

The patient is a local grocery store manager, and her hobbies include playing soccer, hiking, walking, and spending time with family and friends.  She remains active in the winter by swimming at her local gym and walking on the treadmill.  She has been married for 16 years and has a daughter.  Her husband works for an accounting firm and has been there for 20 years.  They utilize the healthcare benefits from her husband’s firm and have never had trouble obtaining healthcare services.  She describes her living environment as safe and frequently walks in the evening with neighbors.  Her diet consists of dark leafy greens, lean proteins such as lean red meat and fish, bananas, apples, water, and mild. 

The patient denies tobacco, alcohol, and illicit drug use.  She takes one supplement daily for bone strengthening and osteoporosis prevention (citing her mom and grandmother had osteoporosis) and OTC medications such as Motrin for her recent ankle injury.  Due to the patient’s history of ankle injuries, education was provided regarding an increased likeliness of such an injury re-occurring; stretching exercises prior to activity and ankle support when participating in physical activities were encouraged.  

Fam Hx:

  1. Father – 81yr old (HTN, Hyperlipidemia)
  2. Mother – 74yr old (HTN, Hx smoking, osteoprorosis Dementia – lives in a long term care facility)
  3. Daughter – 15yr old (No past/current medical history)
  4. Husband – 57yr old (Asthma, HTN, Hx smoking – 16 year pack history)
  5. Maternal Grandmother – 78yr old HTN, osteoporosis
  6. Maternal Grandfather – Passed away 88 years old from heart attack (Hx HTN, hyperlipidemia)
  7. Paternal Grandmother – passed away at 86 years of age of natural causes
  8. Paternal Grandfather – passed away at 96 years of age of COPD (Hx HTN, tobacco use – 60 year pack history)

Immunizations/Vaccinations History:

  1. Influenza Vaccination – annually, last received 10/12/22
  2. COVID 19 Vaccination – 2 doses 9/17/21 and 10/27/21 (did not receive vaccination booster)
  3. DTaP Vaccination – 5 years ago this month 4/26/18

  4. States all vaccinations as child are current and up to date

ROS:

GENERAL:  Patient presents as nervous and is anxious regarding the status of her ankle.

HEENT:  

  1. Head – Denies headaches, dizziness, trauma, lightheadedness, syncope.
  2. Eyes:  Denies changes/loss of vision, blurriness, pain, drainage, or trauma. Patient denies wearing glasses or contacts.
  3. Ears: Denies changes/loss of vision, trauma, discharge, Hx of past ear infection, or pain. Last eye exam was 6 months ago and without significant finding(s).
  4. Nose: Denies drainage, trauma, pain, congestion, bloody nose, or sneezing
  5. Throat:  Denies sore throat, difficulty swallowing, or sore throat. Patient denies changes to voice, hoarseness, or trauma.

SKIN:  Denies episodes of rashes or itching.

CARDIOVASCULAR:  Denies chest pain, pressure, discomfort, or tightness.  Denies shortness of breath (SOB), unusual/easily bruising, swelling of extremities (aside from current injury, patient denies above signs/symptoms).  Denies problems associated with general/overall circulation.

RESPIRATORY:  Denies SOB, difficulty/pain upon inspiration or expiration. Denies cough and states last TB vaccination was at 35 years or age (voluntary as she works with the public).

GASTROINTESTINAL:  Denies anorexia, changes/loss of appetite, nausea/vomiting, or diarrhea. Denies abdominal pain, blood, or changes to stool

GENITOURINARY:  Denies burning/pain/difficulty on urination. Denies current pregnancy, last menstrual period 3/18/23 and have been regular/consistent.  Last visit to gynecologist 2 months ago with unremarkable findings including Pap smear.

NEUROLOGICAL:  No headache, dizziness, syncope, paralysis, ataxia, numbness or tingling in the extremities. No change in bowel or bladder control.

MUSCULOSKELETAL:  Denies back pain, joint pain or stiffness aside from site of injury. Pain level at site of injury (R/ankle) is 8 and described as throbbing and constant that radiates up the leg.  Ice and Motrin for inflammation and pain have slight affect to pain/discomfort levels.

HEMATOLOGIC:  Denies Hx bleeding, bruising, or history of anemia.

LYMPHATICS:  Denies enlarged lymph nodes.  Denies Hx of splenectomy.

PSYCHIATRIC:  Denies Hx of depression, anxiety, or mental illness. Denies sleep disturbances, irritability, difficulty concentrating, and reports overall mood as stable.

ENDOCRINOLOGIC:  Denies sweating or intolerance to heat and cold environments/temperatures. cold or heat intolerance. Denies/has not experienced polyuria, polydipsia, excessive thirst/hunger.

ALLERGIES:  Denies allergies to food, medications, latex, or environmental/seasonal. Denies Hx of rash/hives, eczema, or rhinitis. 

Surgery – denies surgical history

O.

Physical exam:

General: Patient is a 46-year-old female that presents as alert and orientated x3, is dressed appropriately and able to participate in assessment/interview process. Anxiety is present and due to unknown outcome regarding her ankle. Denies fatigue/weakness, fever, or recent changes to weight.

Vital Signs: BP 141/92    HR 99   R20   O2 99%   Temp 98.8F

Height: 64inches   Weight: 128lbs.  BMI: 22.0 (normal weight)

HEENT:

  1. Head:Normocephalic, no indications of masses/tenderness upon palpation.  Scalp is moist and without presence of alopecia (hair evenly distributed).  Cranial nerves intact V and V11 are intact and face symmetrical and without presentation/indication of paralysis or involuntary movement.
  2. Eyes: Sclera white and conjunctivae pink bilaterally.Pupils equal, round, reactive to light, and accommodation (PERRLA); eyes symmetric and placed evenly/appropriately in orbits.  No indication/presentations of redness, discharge/drainage. Cranial nerves 2-4 intact.
  3. Ears: Ear lobes parallel and symmetrical bilaterally and without presence/indication of discharge. Palpation of periorbital sinuses do not indicate presence of tenderness/edema.  Ear drum pinkish/gray, translucent and in neutral position; malleus in oblique position and behind the upper part of drum
  4. Nose: Nose is at midline and without indication of evidence to deviation of bone or cartilage upon palpation; septum is midline and without perforation.  Nasal turbinates and nares are pink, moist, and smooth.  No indication of drainage discharge observed.  Ethmoidal, frontal, maxillary, sphenoidal sinus non-palpable. No indications of obstruction or trauma.
  5. Throat: Tonsils are visible (between the anterior/posterior pillar) with no indication of enlargement; pink buccal mucosa with no indication of lesions, swelling, discoloration, or nodules. Gums are pink and without swelling bleeding, no signs/indication of dental caries or chipped teeth.  Observed as able to swallow without difficulty.
  6. Neck: No indications of masses or lumps, or indications of venous distention.  Patient’s speech is clear and trachea midline, straight, and palpable.  Patient maintains full ROM with absence of nuchal rigidity and non-palpable lymph nodes. 

Respiratory: Chest is symmetrical upon inspirational/expiration.  Breath/lung sounds normal through all fields and without signs/symptoms of fremitus.  Resonance observed throughout all lung fields posterior/anterior and does present with SOB/difficulty breathing.  Slight increase in respirations observed and associated to anxiety and pain level(s).

Cardiovascular:  Heart sounds S1, S2 present and without irregularity; no indication of abnormal rhythms/sounds.  Aortic, pulmonic, mitral, tricuspid sounds present and without abnormality upon auscultation.  No indications of swelling of extremities or signs of cyanosis present.  Capillary refill less than 2 seconds.  Diminished dorsalis pedis pulse of +2 observed in right foot. Dorsalis Pedis pulse to L/ankle +3. Tachycardia and elevated blood pressure present and associated to sustained injury(s). 

Peripheral Vasculature:

  1. Bilateral posterior tibial pulses +3
  2. Bilateral dorsalis pedis pulses – left +2, right +3
  3. Bilateral popliteal pulses +3
  4. Bilateral femoral pulses +3

Skin: Bruising/contusion as evidenced by hematoma at site of injury and warmth indicated over site of injury.  The skin distal from injury is pink, warm and without indication of compromised circulation. 

Musculoskeletal: No indication of L/ankle swelling, bruising, tenderness when palpated.  Presence of +2 Edema to lateral and posterior malleolus of R/ankle over areas of Anterior/posterior inferior tibiofibular (AITFL/PITFL), Anterior/posterior talofibular (ATFL, PTFL) and Calcaneofibular (CFL) ligaments.  Presence of limited/decreased ROM in passive and active ranges of motion including plantar/dorsiflexion, medial/lateral rotation, and eversion/inversion. Erythema observed with tenderness upon palpation. No indication of edema or erythema noted to the ankle. Generalized bruising presents a lateral aspect of right ankle. No indications of deformities or decreased ROM to joints of toes or knees.  Unsteady/limited gait due to R/ankle injury; able to bear weight with associated pain.

Diagnostic results:

External Rotation Test / Kleiger test – Stress test in which the foot is placed in a neutral position with lower leg stabilized; foot is then externally rotated with a presentation of pain, tear of the anterior tibiofibular ligament can be suspected (Larkins et al., 2020). 

Squeeze Test – Stress test involves squeezing the bones test just above the anterior tibiofibular ligament together firmly and slowly and then quickly releasing; the presentation of pain upon release, sprain of anterior tibiofibular ligament can be suspected (2020).

Ultrasound – ankle is moved/manipulated (by healthcare professional) and scanned simultaneously; the image(s) provides information to determine a ligaments level of stability. 

Ct scan – This test allows healthcare professionals to identify fractures that may not be evident in an x-ray; this scan also allows assessment tendons and cartilage and detect damage (tears, ruptures).

X-ray – These images detect dense structures such as bones; as the x-ray is administered, dark lines appear on the image of white bone indicating fracture or break.

Introduction

Emergency departments, urgent care facilities, and orthopedic specialists frequently see patients with ankle injuries; this injury is considered the most common musculoskeletal injury and effects an estimated 2 million individuals annually in the United States (Herzog et al., 2019).   Given its range of motion, abundance of bones, ligaments, and tendons, as well as the fact that the ankles support the weight of the body, the ankle is particularly prone to acute damage. 

The peroneus longus and peroneus brevis muscles and their tendons, the lateral malleolus, calcaneus, talus, fibula bones, as well as the anterior/posterior tibiofibular ligaments, anterior/posterior talofibular ligaments, and calcaneofibular ligament are thr structures that can be associated with lateral ankle pain.  We will address injuries to these tissues caused by acute muscle strains, ligament sprains, or fractures below.

A.

Primary Diagnosis:

Grade 2 Inversion Sprain to Right Ankle

Acute ankle inversion sprains can cause severe short-term disability, reoccurring injuries, and functional instability; factors that determine the type of ankle injury include low or high-energy trauma, location of the foot, and rotational force on the joint and supporting ligamentous structures (Melanson, 2022).  The lateral ligament complex, the medial deltoid ligament, and the syndesmotic ligaments are the three ligamentous systems that sustain, support, and provide stability to the ankle joint in question; ligaments that can be injured due to an inversion sprain include ATFL, CFL, and PTFL. 

The ATFL is the weakest of these three ligaments and is attributed to most inversion sprains (Ball et al., 2018).  There are three grades of sprains; each grade is assigned to the level of injury sustained.  Our patient presents with swelling, bruising, restricted mobility, ROM, tenderness upon palpation, and pain when weight is applied to the area; the main sign or symptom attributed to grade one inversion sprain is the popping sound or sensation that occurs at the time of injury (Sprained Ankle – Symptoms and Causes – Mayo Clinic, 2022). 

Differential Diagnosis:

Avulsion Fracture

An avulsion fracture occurs when a small part of the bone connected to a tendon or ligament becomes separated from the central part of the bone; this happens due to the bone moving in the opposite direction of a ligament or tendon (Avulsion Fracture: What It Is, Causes, Symptoms, Treatment & Recovery, n.d.).  Signs and symptoms can mimic those of inversion sprains and can include swelling, bruising, pain that radiates to other parts of the limb, a popping sound at injury occurrence, and bruising; tenderness over a bony prominence is an indication for this type of injury however, an x-ray or CT scan will be able to differentiate this type of injury from an inversion sprain (Walden, 2022).

Chronic Ankle Instability:

Chronic ankle instability is described as the recurrence of the outer lateral side of the ankle “giving way” due to repeated injury (sprains) of the affected side; symptoms of this condition include pain, tenderness, generalized feeling of weakness at the joint, chronic swelling, repeated “turning of the ankle while active or on uneven surfaces (Chronic Ankle Instability | Tampa General Hospital, n.d.).  These repetitive occurrences are attributed to the site of injury that has not been rehabilitated completely.  Proper therapy is required to strengthen the muscles surrounding the ankle and “retrain” the tissues that impact balance within the ankle (2019).

Peroneal tendon subluxation:

Peroneal subluxation can also be considered a dislocation of a peroneal tendon(s) which are responsible for connecting the peroneal muscles (on outside of calf) to the metatarsal bones located in the foot; the fibrous structure holding these tendons on place is the superior peroneal retinaculum (SPR), this injury isa the result of this structure becomes torn or stretched (Northwestern Medicine, n.d.). 

Symptoms can include swelling, a popping sound at time of injury, instability, reduced ROM, pain is described as sharp in nature and associated around the outside of the ankle when the foot is pointed inwards or rotated inwards when standing; this diagnosis can be ruled out due to the absence of tendon visibly that moves across the bone on the outside of the ankle (Hawes, 2021). 

Anterolateral impingement:

Anterolateral impingement is defined as the entrapment of hypertrophic soft tissue in the lateral gutter; this condition begins when an inversion sprain resulting in a tear to either anterior talofibular, and/or calcaneofibular ligament (Donovan & Rosenberg, 2021).  Symptoms regarding this condition include swelling, tenderness at front of the ankle when palpated, pain associated to front and/or outside of ankle joint; diagnosis can be ruled out due to absence of a decreased ankle ROM when toes are stretched upward towards the shin (Walden, 2022b). 

References

Avulsion Fracture: What It Is, Causes, Symptoms, Treatment & Recovery. (n.d.). Cleveland Clinic. https://my.clevelandclinic.org/health/diseases/21802-avulsion-fracture

Ball, J. W., Dains, J. E., Flynn, J. A., Solomon, B. S., & Stewart, R. W. (2018). Seidel’s Guide to Physical Examination: An Interprofessional Approach. Mosby.

Chronic Ankle Instability | Tampa General Hospital. (n.d.). https://www.tgh.org/institutes-and-services/conditions/chronic-ankle-instability#:~:text=Chronic%20ankle%20instability%20is%20a,when%20you’re%20just%20standing.

Donovan, A., & Rosenberg, Z. S. (2021). MRI of Ankle and Lateral Hindfoot Impingement Syndromes. American Journal of Roentgenology195(3), 595–604. https://doi.org/10.2214/ajr.09.4199

Hawes, E. (2021, March 20). Peroneal Tendon Subluxation | Ankle Subluxation | Simon Moyes [Video]. SIMON MOYES. https://simonmoyes.com/orthopaedic-treatments/ankle-conditions/peroneal-tendon-subluxation/

Herzog, M. M., Kerr, Z. Y., Marshall, S. W., & Wikstrom, E. A. (2019). Epidemiology of Ankle Sprains and Chronic Ankle Instability. Journal of Athletic Training54(6), 603–610. https://doi.org/10.4085/1062-6050-447-17

Larkins, L., Baker, R. T., & Baker, J. G. (2020). Physical Examination of the Ankle: A Review of the Original Orthopedic Special Test Description and Scientific Validity of Common

Tests for Ankle Examination. Archives of Rehabilitation Research and Clinical Translation2(3), 100072. https://doi.org/10.1016/j.arrct.2020.100072Links to an external site.

Melanson, S. W. (2022, May 29). Acute Ankle Sprain. StatPearls – NCBI Bookshelf. https://www.ncbi.nlm.nih.gov/books/NBK459212/Links to an external site.

Northwestern Medicine. (n.d.). Peroneal Tendon Subluxation. https://www.nm.org/conditions-and-care-areas/orthopaedics/foot-and-ankle/peroneal-tendon-subluxation

Sprained ankle – Symptoms and causes – Mayo Clinic. (2022, August 11). Mayo Clinic. https://www.mayoclinic.org/diseases-conditions/sprained-ankle/symptoms-causes/syc-20353225

Walden, M. (2022). Ankle Avulsion Fracture. Sportsinjuryclinic.nethttps://www.sportsinjuryclinic.net/sport-injuries/ankle-pain/acute-ankle-injuries/ankle-avulsion-fractureLinks to an external site.

Walden, M. (2022b). Ankle Impingement. Sportsinjuryclinic.net. https://www.sportsinjuryclinic.net/sport-injuries/ankle-pain/anterior-ankle-pain/ankle-impingement

Note: For this Discussion, you are required to complete your initial post before you will be able to view and respond to your colleagues’ postings. Begin by clicking on the “Post to Discussion Question” link and then select “Create Thread” to complete your initial post. Remember, once you click on Submit, you cannot delete or edit your own posts, and you cannot post anonymously. Please check your post carefully before clicking on Submit!

Read a selection of your colleagues’ responses.

Episodic/Focused SOAP Note Template

Patient Information:

PH, 15-years-old, Male, Filipino

S.

CC: Knee pain

HPI: PH, 15-year-old Filipino male presents with bilateral knee pain for over a week. PH describes the pain as dull with occasional “clicking” or “catching” in one or both knees. PH reports that the pain started a couple weeks after basketball season started this year. PH stated he had pain similar to this last spring during track when he started competing in long jump. PH reports that it hurts more after practice than it does after a game stating, “coach has me doing extra running and jumping drills, he’s really hard on us.”

Location: Knees, under patella

Onset: A week ago

Character: dull

Associated signs and symptoms: occasional clicking of one/both knees, “catching” sensation

Timing: “after practice”

Exacerbating/relieving factors: track and basketball practice make it worse; ibuprofen and ice/heat help make it ache less

Severity: 6/10 pain scale

Current Medications:

Ibuprofen 200mg PO after practice

Multivitamin (OTC) PO daily

Allergies:

Denies food or environmental allergies.

Adhesives- rash at site

Tylenol- nausea

PMHx:

Current and up to date on all immunizations, influenza vaccine received this season, but did not receive COVID-19 vaccines.

Tonsillectomy and adenoidectomy- 2012

Fractured Ulna- 2020 from basketball injury

Reports several sprained ankles from basketball and track.

Denies concussions or previous knee injuries.

Soc Hx:

Freshman in high school, track and field athlete (long jumper), and JV basketball player (small forward). Lives with parents and younger brother. Denies tobacco, alcohol, or illicit drug use. Denies sexual activity, advises he has a girlfriend. PH reports always wearing a seatbelt. PH plays golf with friends outside of school and enjoys playing Xbox on weekends with his friends from the track team. PH reports he has several friends that support him at school and family that encourage him in academics and sports. PH advises he feels a lot of pressure from basketball coach to “be the best”.

Fam Hx:

Mother: HTN, Hyperlipidemia, Depression

Father: DM2, HTN, Hyperlipidemia

Brother: No known history.

MGM: HTN

MGF: No information available

PGM: HTN, Hyperlipidemia

PGF: Deceased at 54 from MI

ROS:

Example of Complete ROS:

GENERAL:  No weight loss, fever, or chills. Reports general weakness after practices and games. Reports academic and athletic success.

HEENT:  Eyes:  No visual loss, blurred vision, double vision or yellow sclerae. Ears: No hearing loss or tinnitus. Nose: No sneezing, congestion, loss of smell, runny nose, or epistaxis. Throat: No sore throat, erythema, or lesions.

SKIN:  No rash or itching. Reports having occasional acne on forehead. Reports underarm hair.

CARDIOVASCULAR:  No chest pain, chest pressure or chest discomfort. No palpitations or edema.

RESPIRATORY:  No shortness of breath, cough, or sputum. Reports infrequent, unproductive cough after running sprints during basketball practice.

GASTROINTESTINAL:  Denies anorexia, nausea, vomiting or diarrhea. No abdominal pain or blood. Last BM 1/16/23 no melena, constipation, or loose stool.

GENITOURINARY:  Denies burning or pain with urination, frequency, or nocturia.  

NEUROLOGICAL:  No dizziness, syncope, paralysis, ataxia, numbness or tingling in the extremities. Reports headaches after studying all night for tests.

MUSCULOSKELETAL:  Reports dull bilateral knee pain. Ankle pain previously with sprain—currently resolved. Denies immobility.

HEMATOLOGIC:  No anemia or bleeding. Identifies bruise on right shin from exercise injury.

LYMPHATICS:  No enlarged nodes. No history of splenectomy.

PSYCHIATRIC:  No history of depression or anxiety. Reports feeling pressure from basketball coach to “be the best.” Reports a supportive social and family group.

ENDOCRINOLOGIC:  No reports of cold or heat intolerance. No polyuria or polydipsia. Reports sweating more than last year and must apply deodorant again before practice.

ALLERGIES:  No history of asthma, environmental or food allergies. Acetaminophen- nausea. Adhesives (tape)- rash at site.

O.

Physical exam:

VS: BP 115/64, HR 80, RR 16, O2 98%, 36.8 C, 65 inches, 54.4kg, BMI 20

GENERAL: PH is dressed appropriately and well groomed. PH able to maintain erect sitting position for examination and appropriate historian for current chief complaint, requires assistance from mother on allergies and past medical history.

NEUROLOGICAL: Alert and oriented. Cranial nerves intact. Upper and Lower extremity strength equal bilaterally. Reflexes 2+.

SKIN: Acne present near hairline of forehead. No scaring, lesions, rashes, or moles present. Appropriate for ethnicity. Hair pattern presenting at underarms. Nails are short without brittle texture, pitting, or ridging.

HEENT: Head- symmetric. Ears: symmetric bilateral, negative tenderness and discharge, eardrum pearly grey, and no erythema. Eye: PERRLA 3/2 brisk equal response, negative nystagmus, white sclera, brown iris, and eye lids equal. 20/20 vision using Snellen chart. Nose: nares equal, no erythema in nasal cavity, no postnasal drip, patent turbinates, no polyps, PH can identify smells presented.

NECK: No palpable/enlarged lymph nodes. Trachea midline. Appropriate ROM bilateral.

RESPIRATORY: No SOB. Breath sounds clear a/p in all lobes. No adventitious breath sounds. Chest expansion symmetric with inhalation and exhalation. Work of breath appropriate for examination, no dyspnea noted.

CARDIOVASCULAR: S1, S2 heard, rate and rhythm regular. No carotid, renal, or aortic bruits identified. Extremities color appropriate for ethnicity and warm. Radial and dorsalis pedis pulses 2+.

ABDOMEN: Symmetric, normoactive bowel sounds all quadrants, no palpable masses, no tenderness or guarding.

GENITALIA: Minimal pubic hair noted, uncircumcised, no tenderness at penis or scrotum, no masses identified.

MUSCULOSKELETAL: ROM of all extremities appropriate, stiffness and guarding of knees occurs when transitioning from sitting to standing. Clicking noise noted when patient stood. No joint swelling or redness. No kyphosis, lordosis, or scoliosis. Grade 5 muscle strength. Obvious discomfort noted with initial transition from sitting to standing and stepping up on step. Q angle 14°.

Diagnostic results:

Ultrasound- This a non-invasive test that can provide dynamic structure images of the knee to confirm or eliminate possible diagnoses (Santana & Sherman, 2020).

MRI- Can be used to identify patellar tendon abnormalities as well as osseous and soft tissue injuries (Nacey et al., 2017). Ultrasounds used in conjunction with MRI’s can both eliminate diagnoses, but also confirm.

Erythrocyte Sedimentation Rate (ESR)- this lab test measures the sedimentation rate of red blood cells to detect inflammation. This can be used to diagnose juvenile arthritis (JA) (Daines et al., 2019).

CBC- this lab test would be used to identify an increase in WBC which can identify inflammation as well but is not diagnostic (Dains et al., 2019).

X-ray- a four view radiograph image (a/p, lateral, and skyline views are useful to eliminate JA as well as determine if there are osseous abnormalities (Santana & Sherman, 2020).

A.

Differential Diagnoses

  1. Patellar Tendinitis – According to Santana & Sherman (2020), patellar tendinitis (also

known as “jumper’s knee”) is a result of overuse and stress on the patellar tendon and quadriceps. History and signs of patellar tendinitis include overuse of the jumping or running motion and symptoms include dull pain at the patella and clicking can accompany the pain (Dains et al., 2019). Due to PH recent overuse of patellar tendons and quadriceps with long jumping in the spring and currently overworked during basketball season has caused the patient to present with dull knee pain and clicking and catching of knees making patellar tendinitis most likely. Ruling out other diseases by utilizing US, MRI, X-ray, CBC, and ESR since there is no definitive test to confirm patellar tendinitis (Santana & Sherman, 2020).

  1. Chondromalacia of the Patella – Dains et al. (2019) is caused by trauma to the patella, misalignment, or anatomical abnormalities. Four-view radiography should be used to rule out JA. Chondromalacia pain occurs with activity rather a result of activity making it less likely to be the diagnosis since PH pain occurs after strenuous activity and there is no pain on patella palpitation. Habusta & Griffin (2020) also state that this is more prominent is young female adolescents, not males.
  2. Juvenile Arthritis – Dains et al. (2019) describe history of JA as joint stiffness and pain, fatigue, weight loss, and refusal to walk with symptoms being limited ROM, bilateral joint pain, rash, fever, and joint swelling. PH presents with painful knees bilaterally, but does not present with fever, joint swelling, or rash. A CBC would demonstrate increased WBC, ESR would be positive for antinuclear antibodies and rheumatoid factor. Most JA diagnosis occurs before 16-years-old, it does not appear this patient is likely to have JA, but should not be eliminated until diagnostic tests and labs can be completes.
  3. Bursitis – Bursitis is caused by chronic overuse resulting in local tenderness and swelling, limited joint movement, and muscle weakness (Dains et al., 2019). US and MRI testing combined can be used to identify bursitis and the depth and severity of inflammation (Williams & Sternard, 2019). PH’s injury is acute and is not from chronic overuse; however should not be eliminated until an US and MRI can rule out bursitis. This is not a primary diagnosis due to lack of symptoms mirroring what the patient is presenting with.
  4. Patellar Maltracking – “Patellar maltracking occurs as a result of an imbalance in the dynamic relationship between the patella and trochlea. This is often secondary to an underlying structural abnormality,” (Jibri et al., 2019). Imaging used to diagnose maltracking includes MRI to note subtle changes in dislocation of the patella; however, without any documented history of dislocation it makes it difficult to diagnose.
  5.  

This section is not required for the assignments in this course (NURS 6512) but will be required for future courses.

When assessing a patient getting a full history is essential to assist in diagnosing a patient or ordering the most appropriate tests and imaging to rule out possible diagnoses. This patient case study only described the pain and stated his age, current activities were up to us to fill in which guides the differential diagnosis. If this patient did not demonstrate overuse of the patella, it would make some differential diagnoses unlikely. Based on the information provided and filled in determines the overall diagnostic tests and diagnosis. Listed in the subjective and objective data is both information provided and information needed to create possible differential diagnoses.

References

Dains, J. E., Baumann, L. C., & Scheibel, P. (2019). Advanced health assessment and clinical diagnosis in primary care (6th ed.). St. Louis, MO: Elsevier Mosby.

Habusta, S. F., & Griffin, E. E. (2020). Chondromalacia Patella. PubMed; StatPearls Publishing. https://www.ncbi.nlm.nih.gov/books/NBK459195/

Links to an external site.

Jibri, Z., Jamieson, P., Rakhra, K. S., Sampaio, M. L., & Dervin, G. (2019). Patellar maltracking: an update on the diagnosis and treatment strategies. Insights into imaging10(1), 65. https://doi.org/10.1186/s13244-019-0755-1

Nacey, N. C., Geeslin, M. G., Miller, G. W., & Pierce, J. L. (2017). Magnetic resonance imaging of the knee: An overview and update of conventional and state of the art imaging. Journal of magnetic resonance imaging : JMRI45(5), 1257–1275. https://doi.org/10.1002/jmri.25620

Santana, J. A., & Sherman, A. l. (2020). Jumpers Knee. PubMed; StatPearls Publishing. https://www.ncbi.nlm.nih.gov/books/NBK532969/

‌ Williams, C. H., & Sternard, B. T. (2019). Bursitis. Nih.gov; StatPearls Publishing. https://www.ncbi.nlm.nih.gov/books/NBK513340/

By Day 6

Respond to at least two of your colleagues on 2 different days who were assigned different case studies than you. Analyze the possible conditions from your colleagues’ differential diagnoses. Determine which of the conditions you would reject and why. Identify the most likely condition, and justify your reasoning.

Submission and Grading Information

Grading Criteria

To access your rubric:

Week 8 Discussion Rubric

Post by Day 3 and Respond by Day 6

To participate in this Discussion:

Week 8 Discussion

 

Assignment: Assessing the Skin, Hair, Nails, and HEENT

In preparation for the Head-to-Toe Physical Assessment Video due in Week 10, you will videotape yourself conducting an assessment of the skin, hair, nails, and HEENT this week.

To prepare:

  • Arrange an appropriate time and setting with your volunteer “patient” to perform a skin, hair, nails, and HEENT examination.
  • Submit your volunteer’s Video Release form prior to the exam.
  • Download and review the Skin, Hair, and Nails and HEENT rubric provided in this week’s Learning Resources.
  • Ensure that you have the appropriate lighting and equipment to perform the examination.

To complete:

  • Record yourself performing the skin, hair, and nails, and HEENT physical examination. Be sure to cover all of the areas listed in the rubric and to use any equipment appropriately. This Assignment is due by Day 7 of Week 8. Submit your video using the Kaltura Mashup tool accessible through the Assignment submission link provided.

To submit your completed Video Assignment(s), do the following:

If you have not already done so, click on the Week 8 Assignment link. Once you have clicked on the link, click on the Write Submission button to turn on the Content Editor toolbar. Next, fill in the Submission field with any pertinent information. Attach your Assignment file by clicking on the Mashup button on the text editor menu bar and select Kaltura Media. Then find the media file you saved as “WK8Assgn+first initial+last name” and click on Open. Add any appropriate comments pertaining to your Assignment(s) in the Comments field. Be sure to attach all your video assignments. Finally, click on the Submit button to turn in your Assignment(s) for review.

For additional details on using the Kaltura Media mashup tool, please refer to the Kaltura Media Uploader page located in the course navigation menu.

By Day 7

This assignment is due.

Submission and Grading Information

Grading Criteria

To access your rubric:

Week 8 Assignment Rubric

Check Your Assignment Draft for Authenticity

To check your Assignment draft for authenticity:

Submit your Week 8 Assignment draft and review the originality report.

Submit Your Assignment by Day 7

To submit your Assignment:

Week 8 Assignment

 

Assignment 2 (Optional): Practice Assessment: Musculoskeletal Examination

A description of symptoms alone is not enough to form an accurate diagnosis of musculoskeletal conditions. Before forming a diagnosis, advanced practice nurses need to perform a physical examination. Although the musculoskeletal examination is relatively simple, it still needs to be performed multiple times before it can be mastered.

In preparation for the Head-to-Toe Physical Assessment Video due in Week 10, it is recommended that you practice performing a musculoskeletal examination this week.

Note: This is an optional practice physical assessment. You do not have to capture a video of this assessment, as no submission is required.

To prepare:

  • Arrange an appropriate time and setting with your volunteer “patient” to perform a musculoskeletal examination.
  • Download and review the Musculoskeletal Checklist provided in this week’s Learning Resources.

To complete:

  • Perform the musculoskeletal examination. Be sure to cover all of the areas listed in the checklist.

I enjoyed reading your post! In your episodic/SOAP note, you gave detailed information and painted a “realistic picture” of the patient. Recently, a medical doctor told me, “our bodies give us warning signs when it’s in distress. When the body is in distress, it tries to repair the issue. If the body does not repair the issue on its own, it’s up to healthcare professionals to figure out the etiology and treatment.”

Musculoskeletal conditions comprise one of the leading causes of severe long-term pain in patients. The musculoskeletal system is an elaborate system of interconnected levers that provides the body with support and mobility. Because of the interconnectedness of the musculoskeletal system, identifying the causes of pain can be challenging. Accurately interpreting the cause of musculoskeletal pain requires an assessment process informed by patient history and physical exams.

Case Study 3

In your assigned case study, the patient, PH,  is a 15-year-old Filipino boy with complaints of bilateral knee pain for over a week. He describes the pain as dull intermittent “clicking” or “catching in one or both knees. PH reports that the pain started a couple weeks after basketball season started this year.

PH stated he had pain similar to this last spring during track when he started competing in long jump. PH reports that it hurts more after practice than it does after a game stating, “coach has me doing extra running and jumping drills, he’s really hard on us.” He has a history of an ulnar fracture and multiple sprained ankles from basketball and track, but no previous knee injuries.

Patellar injury differential diagnosis- Patellar Tendonitis

Your differential diagnosis were Patellar Tendonitis, Chondromalacia of the Patella, Juvenile Arthritis, Bursitis, and Patellar Maltracking. Agreeably so, I believe Patellar Tendonitis is the primary diagnosis. Your description of the condition is best with PH reported signs and symptoms. Chronic inflammation, such as patellar tendonitis, leads to a weakened tendon and can increase the likelihood of tendon rupture. Certain medical conditions can lead to an overall weakened tendon and can also predispose an individual to tendon rupture such as patellar degeneration, overuse injury, and previous injury (Hsu & Siwiec, 20121). 

Patellar injury differential diagnosis- Chondromalacia of the Patella 

Chondromalacia of the Patella occurs with activity rather than a result of the activity. Chondromalacia patella (CMP) is when the posterior articular surface of the patella starts losing its density when in a healthy state and turns to be softer with subsequent tearing, fissuring, and erosion of the hyaline cartilage (Habusta et al., 2021). You stated that the condition is found more in women than men. According to (Habusta et al., 2021), CMP is more common in women than men and this is attributed to increased Q angles in women. Therefore, this will be a least likely primary diagnosis for PH since he is male gender. 

Patellar injury differential diagnosis- Juvenile Arthritis

Juvenile Arthritis (JA), isn’t a specific condition. It is a broad term that describes numerous rheumatoid conditions in children. Similar to arthritis observed in adults, pathogenesis involves autoimmune and autoinflammatory mechanisms (Martini et al., 2022). Agreeably so, the majority of JA conditions are diagnosed at age 16 and older. One with JA can exhibit a fever, joint inflammation, swelling, pain and tenderness, but some types of JA have few or no joint symptoms or only affect the skin and internal organs (Arthritis Foundation, 2021). As you stated, it is least likely that PH has JA, but should not be completely eliminated until ruled out by further testing. 

Patellar injury differential diagnosis- Bursitis

Bursitis does require treatment by a physician. The olecranon and prepatellar bursae are the most often involved sites, as their superficial location exposes them to injury. Among patients with bursitis, 80% are males aged 40 to 80 years who constitute the population most exposed to trauma and micro trauma during manual labor or recreational activities (Lormeau et al., 2019). PH unlikely has Bursitis due to the big gap in age and presenting symptoms. Therefore, I would eliminate this differential diagnosis. 

Patellar injury differential diagnosis- Patellar Maltracking

Your last differential diagnosis was Patellar Maltracking. Patellar Maltracking refers to the dynamic relationship between the patella and trochlea during knee motion. Patellar maltracking occurs as a result of imbalance of this relationship often secondary to anatomic morphologic abnormality. Usually, young individuals, particularly women, suffer the consequences of this disorder (Jibri et al., 2019). 

References

Arthritis Foundation. (2023). Juvenile Arthritis (JA). Retrieved January 17, 2023 https://www.arthritis.org/diseases/juvenile-arthritis

Habusta, S. F., Coffey, R., Ponnarasu, S., & Griffin, E. E. (2021). Chondromalacia patella. In StatPearls [Internet]. StatPearls Publishing.

Hsu, H., & Siwiec, R. M. (2021). Patellar tendon rupture. In StatPearls [Internet]. StatPearls Publishing.

Jibri, Z., Jamieson, P., Rakhra, K. S., Sampaio, M. L., & Dervin, G. (2019). Patellar maltracking: an update on the diagnosis and treatment strategies. Insights into imaging10(1), 1-11.

Lormeau, C., Cormier, G., Sigaux, J., Arvieux, C., & Semerano, L. (2019). Management of septic bursitis. Joint Bone Spine86(5), 583-588.Martini, A., Lovell, D. J., Albani, S., Brunner, H. I., Hyrich, K. L., Thompson, S. D., & Ruperto, N. (2022). Juvenile idiopathic arthritis. Nature Reviews Disease Primers8(1), 1-18.

Martini, A., Lovell, D. J., Albani, S., Brunner, H. I., Hyrich, K. L., Thompson, S. D., & Ruperto, N. (2022). Juvenile idiopathic arthritis. Nature Reviews Disease Primers8(1), 1-18.

Rubric Detail

Select Grid View or List View to change the rubric’s layout.

Content

  Outstanding Performance Excellent Performance Competent Performance Proficient Performance Room for Improvement
Main Posting:
Response to the discussion question is reflective with critical analysis and synthesis representative of knowledge gained from the course readings for the module and current credible sources.
 
Points Range: 44 (44%) – 44 (44%)

Thoroughly responds to the discussion question(s)

is reflective with critical analysis and synthesis representative of knowledge gained from the course readings for the module and current credible sources.

supported by at least 3 current, credible sources

 
Points Range: 40 (40%) – 43 (43%)

Responds to the discussion question(s)

is reflective with critical analysis and synthesis representative of knowledge gained from the course readings for the module.

75% of post has exceptional depth and breadth

supported by at least 3 credible references

 
Points Range: 35 (35%) – 39 (39%)

Responds to most of the discussion question(s)

is somewhat reflective with critical analysis and synthesis representative of knowledge gained from the course readings for the module.

50% of post has exceptional depth and breadth

supported by at least 3 credible references

 
Points Range: 31 (31%) – 34 (34%)

Responds to some of the discussion question(s)

one to two criteria are not addressed or are superficially addressed

is somewhat lacking reflection and critical analysis and synthesis

somewhat represents knowledge gained from the course readings for the module.

post is cited with fewer than 2 credible references

 
Points Range: 0 (0%) – 30 (30%)

Does not respond to the discussion question(s)

lacks depth or superficially addresses criteria

lacks reflection and critical analysis and synthesis

does not represent knowledge gained from the course readings for the module.

contains only 1 or no credible references

Main Posting:
Writing
 
Points Range: 6 (6%) – 6 (6%)

Written clearly and concisely

Contains no grammatical or spelling errors

Fully adheres to current APA manual writing rules and style

 
Points Range: 5.5 (5.5%) – 5.5 (5.5%)

Written clearly and concisely

May contain one or no grammatical or spelling error

Adheres to current APA manual writing rules and style

 
Points Range: 5 (5%) – 5 (5%)

Written concisely

May contain one to two grammatical or spelling error

Adheres to current APA manual writing rules and style

 
Points Range: 4.5 (4.5%) – 4.5 (4.5%)

Written somewhat concisely

May contain more than two spelling or grammatical errors

Contains some APA formatting errors

 
Points Range: 0 (0%) – 4 (4%)

Not written clearly or concisely

Contains more than two spelling or grammatical errors

Does not adhere to current APA manual writing rules and style

Main Posting:
Timely and full participation
 
Points Range: 10 (10%) – 10 (10%)

Meets requirements for timely and full participation

posts main discussion by due date

 
Points Range: 0 (0%) – 0 (0%)
NA
 
Points Range: 0 (0%) – 0 (0%)
NA
 
Points Range: 0 (0%) – 0 (0%)
NA
 
Points Range: 0 (0%) – 0 (0%)
Does not meet requirement for full participation
First Response:

Post to colleague’s main post that is reflective and justified with credible sources.

 
Points Range: 9 (9%) – 9 (9%)

Response exhibits critical thinking and application to practice settings

responds to questions posed by faculty

the use of scholarly sources to support ideas demonstrates synthesis and understanding of learning objectives

 
Points Range: 8.5 (8.5%) – 8.5 (8.5%)
Response exhibits critical thinking and application to practice settings
 
Points Range: 7.5 (7.5%) – 8 (8%)
Response has some depth and may exhibit critical thinking or application to practice setting
 
Points Range: 6.5 (6.5%) – 7 (7%)
Response is on topic, may have some depth
 
Points Range: 0 (0%) – 6 (6%)
Response may not be on topic, lacks depth
First Response:
Writing
 
Points Range: 6 (6%) – 6 (6%)

Communication is professional and respectful to colleagues

Response to faculty questions are fully answered if posed

Provides clear, concise opinions and ideas that are supported by two or more credible sources

Response is effectively written in Standard Edited English

 
Points Range: 5.5 (5.5%) – 5.5 (5.5%)

Communication is professional and respectful to colleagues

Response to faculty questions are answered if posed

Provides clear, concise opinions and ideas that are supported by two or more credible sources

Response is effectively written in Standard Edited English

 
Points Range: 5 (5%) – 5 (5%)

Communication is mostly professional and respectful to colleagues

Response to faculty questions are mostly answered if posed

Provides opinions and ideas that are supported by few credible sources

Response is written in Standard Edited English

 
Points Range: 4.5 (4.5%) – 4.5 (4.5%)

Responses posted in the discussion may lack effective professional communication

Response to faculty questions are somewhat answered if posed

Few or no credible sources are cited

 
Points Range: 0 (0%) – 4 (4%)

Responses posted in the discussion lack effective

Response to faculty questions are missing

No credible sources are cited

First Response:
Timely and full participation
 
Points Range: 5 (5%) – 5 (5%)

Meets requirements for timely and full participation

posts by due date

 
Points Range: 0 (0%) – 0 (0%)
NA
 
Points Range: 0 (0%) – 0 (0%)
NA
 
Points Range: 0 (0%) – 0 (0%)
NA
 
Points Range: 0 (0%) – 0 (0%)
Does not meet requirement for full participation
Second Response:
Post to colleague’s main post that is reflective and justified with credible sources.
 
Points Range: 9 (9%) – 9 (9%)

Response exhibits critical thinking and application to practice settings * responds to questions posed by faculty

the use of scholarly sources to support ideas demonstrates synthesis and understanding of learning objectives

 
Points Range: 8.5 (8.5%) – 8.5 (8.5%)
Response exhibits critical thinking and application to practice settings
 
Points Range: 7.5 (7.5%) – 8 (8%)
Response has some depth and may exhibit critical thinking or application to practice setting
 
Points Range: 6.5 (6.5%) – 7 (7%)
Response is on topic, may have some depth
 
Points Range: 0 (0%) – 6 (6%)
Response may not be on topic, lacks depth
Second Response:
Writing
 
Points Range: 6 (6%) – 6 (6%)

Communication is professional and respectful to colleagues

Response to faculty questions are fully answered if posed

Provides clear, concise opinions and ideas that are supported by two or more credible sources

Response is effectively written in Standard Edited English

 
Points Range: 5.5 (5.5%) – 5.5 (5.5%)

Communication is professional and respectful to colleagues

Response to faculty questions are answered if posed

Provides clear, concise opinions and ideas that are supported by two or more credible sources

Response is effectively written in Standard Edited English

 
Points Range: 5 (5%) – 5 (5%)

Communication is mostly professional and respectful to colleagues

Response to faculty questions are mostly answered if posed

Provides opinions and ideas that are supported by few credible sources

Response is written in Standard Edited English

 
Points Range: 4.5 (4.5%) – 4.5 (4.5%)

Responses posted in the discussion may lack effective professional communication

Response to faculty questions are somewhat answered if posed

Few or no credible sources are cited

 
Points Range: 0 (0%) – 4 (4%)

Responses posted in the discussion lack effective

Response to faculty questions are missing

No credible sources are cited

Second Response:
Timely and full participation
 
Points Range: 5 (5%) – 5 (5%)

Meets requirements for timely and full participation

Posts by due date

 
Points Range: 0 (0%) – 0 (0%)
NA
 
Points Range: 0 (0%) – 0 (0%)
NA
 
Points Range: 0 (0%) – 0 (0%)
NA
 
Points Range: 0 (0%) – 0 (0%)
Does not meet requirement for full participation
Total Points: 100
 

Week 8: Assessment of the Musculoskeletal System

A 46-year-old man walks into a doctor’s office complaining of tripping over doorways more frequently. He does not know why. What could be the causes of this condition?

Without the ability to use the complex structure and range of movement afforded by the musculoskeletal system, many of the physical activities individuals enjoy would be curtailed. Maintaining the health of the musculoskeletal system will ensure that patients live a life of full mobility. One of the most basic steps that can be taken to preserve the health of the musculoskeletal system is to perform an assessment.

This week, you will explore how to assess the musculoskeletal system.

Learning Objectives

Students will:
  • Evaluate abnormal musculoskeletal findings
  • Apply concepts, theories, and principles relating to health assessment techniques and diagnoses for the musculoskeletal system
  • Evaluate musculoskeletal X-Ray imaging

Learning Resources

Required Readings (click to expand/reduce)

Ball, J. W., Dains, J. E., Flynn, J. A., Solomon, B. S., & Stewart, R. W. (2019). Seidel’s guide to physical examination: An interprofessional approach (9th ed.). St. Louis, MO: Elsevier Mosby.

  • Chapter 4, “Vital Signs and Pain Assessment” (Previously read in Week 6)
  • Chapter 22, “Musculoskeletal System”

    This chapter describes the process of assessing the musculoskeletal system. In addition, the authors explore the anatomy and physiology of the musculoskeletal system.

Dains, J. E., Baumann, L. C., & Scheibel, P. (2019). Advanced health assessment and clinical diagnosis in primary care (6th ed.). St. Louis, MO: Elsevier Mosby.

Credit Line: Advanced Health Assessment and Clinical Diagnosis in Primary Care, 6th Edition by Dains, J.E., Baumann, L. C., & Scheibel, P. Copyright 2019 by Mosby. Reprinted by permission of Mosby via the Copyright Clearance Center.

Chapter 22, “Lower Extremity Limb Pain”
This chapter outlines how to take a focused history and perform a physical exam to determine the cause of limb pain. It includes a discussion of the most common tests used to assess musculoskeletal disorders.

Chapter 24, “Low Back Pain (Acute)”
The focus of this chapter is the identification of the causes of lower back pain. It includes suggested physical exams and potential diagnoses.

Sullivan, D. D. (2019). Guide to clinical documentation (3rd ed.). Philadelphia, PA: F. A. Davis.

  • Chapter 2, “The Comprehensive History and Physical Exam” (“Muscle Strength Grading”) (Previously read in Weeks 1, 2, 3, 4, and 5)
  • Chapter 3, “SOAP Notes”

    This section explains the procedural knowledge needed to perform musculoskeletal procedures.

Note: Download this Student Checklist and Abdomen Key Points to use during your practice abdominal examination.

Ball, J. W., Dains, J. E., Flynn, J. A., Solomon, B. S., & Stewart, R. W. (2019). Musculoskeletal system: Student checklist. In Seidel’s guide to physical examination: An interprofessional approach (9th ed.). St. Louis, MO: Elsevier Mosby.

Credit Line: Seidel’s Guide to Physical Examination, 9th Edition by Ball, J. W., Dains, J. E., Flynn, J. A., Solomon, B. S., & Stewart, R. W. Copyright 2019 by Elsevier Health Sciences. Reprinted by permission of Elsevier Health Sciences via the Copyright Clearance Center.

Ball, J. W., Dains, J. E., Flynn, J. A., Solomon, B. S., & Stewart, R. W. (2019). Musculoskeletal system: Key points. In Seidel’s guide to physical examination: An interprofessional approach (9th ed.). St. Louis, MO: Elsevier Mosby.

Credit Line: Seidel’s Guide to Physical Examination, 9th Edition by Ball, J. W., Dains, J. E., Flynn, J. A., Solomon, B. S., & Stewart, R. W. Copyright 2019 by Elsevier Health Sciences. Reprinted by permission of Elsevier Health Sciences via the Copyright Clearance Center.

Foster, N. E., Anema, J. R., Cherkin, D., Chou, R., Cohen, S. P., Gross, D. P., Ferreira, P. H., Fritz, J. M., Koes, B. W., Peul, W., Turner, J. A., Maher, C. G., Buchbinder, R., Hartvigsen, J., Cherkin, D., Foster, N. E., Maher, C. G., Underwood, M., van Tulder, M., . . . Woolf, A. (2018). Prevention and treatment of low back pain: evidence, challenges, and promising directions. The Lancet, 391(10137), 2368–2383. https://doi.org/10.1016/s0140-6736(18)30489-6

Hicks, C., Levinger, P., Menant, J. C., Lord, S. R., Sachdev, P. S., Brodaty, H., & Sturnieks, D. L. (2020). Reduced strength, poor balance and concern about falls mediate the relationship between knee pain and fall risk in older people. BMC Geriatrics, 20(1), 94. https://doi.org/10.1186/s12877-020-1487-2

Document: Episodic/Focused SOAP Note Exemplar (Word document)

Document: Episodic/Focused SOAP Note Template (Word document)

Optional Resource

LeBlond, R. F., Brown, D. D., & DeGowin, R. L. (2020). DeGowin’s diagnostic examination (11th ed.). New York, NY: McGraw Hill Medical.

  • Chapter 13, “The Spine, Pelvis, and Extremities” 

    In this chapter, the authors explain the physiology of the spine, pelvis, and extremities. The chapter also describes how to examine the spine, pelvis, and extremities.

Required Media (click to expand/reduce)

Musculoskeletal System – Week 8 (12m)

Online media for Seidel’s Guide to Physical Examination

In addition to this week’s resources, it is highly recommended that you access and view the resources included with the course text, Seidel’s Guide to Physical Examination. Focus on the videos and animations in Chapter 21 that relate to the assessment of the musculoskeletal system. Refer to the Week 4 Learning Resources area for access instructions on https://evolve.elsevier.com/ 

Marquis, P. (2019, April 4). Orthopedic knee evaluation with Paul Marquis PT [Video]. YouTube. https://www.youtube.com/watch?v=YVx4BepjjiY&feature=youtu.be


Discussion: Assessing Musculoskeletal Pain

Photo Credit: Getty Images/Fotosearch RF

The body is constantly sending signals about its health. One of the most easily recognized signals is pain. Musculoskeletal conditions comprise one of the leading causes of severe long-term pain in patients. The musculoskeletal system is an elaborate system of interconnected levers that provides the body with support and mobility. Because of the interconnectedness of the musculoskeletal system, identifying the causes of pain can be challenging. Accurately interpreting the cause of musculoskeletal pain requires an assessment process informed by patient history and physical exams.

In this Discussion, you will consider case studies that describe abnormal findings in patients seen in a clinical setting.

To prepare:

  • By Day 1 of this week, you will be assigned to one of the following specific case studies for this Discussion. Please see the “Course Announcements” section of the classroom for your assignment from your Instructor.
  • Your Discussion post should be in the Episodic/Focused SOAP Note format rather than the traditional narrative style Discussion posting format. Refer to Chapter 2 of the Sullivan text and the Episodic/Focused SOAP Template in the Week 5 Learning Resources for guidance. Remember that all Episodic/Focused SOAP notes have specific data included in every patient case.
  • Review the following case studies:

Case 1: Back Pain

Photo Credit: University of Virginia. (n.d.). Lumbar Spine Anatomy [Photograph]. Retrieved from http://www.med-ed.virginia.edu/courses/rad/ext/5lumbar/01anatomy.html. Used with permission of University of Virginia.

A 42-year-old male reports pain in his lower back for the past month. The pain sometimes radiates to his left leg. In determining the cause of the back pain, based on your knowledge of anatomy, what nerve roots might be involved? How would you test for each of them? What other symptoms need to be explored? What are your differential diagnoses for acute low back pain? Consider the possible origins using the Agency for Healthcare Research and Quality (AHRQ) guidelines as a framework. What physical examination will you perform? What special maneuvers will you perform?

Case 2: Ankle Pain

Photo Credit: University of Virginia. (n.d.). Lateral view of ankle showing Boehler’s angle [Photograph]. Retrieved from http://www.med-ed.virginia.edu/courses/rad/ext/8ankle/01anatomy.html. Used with permission of University of Virginia.

A 46-year-old female reports pain in both of her ankles, but she is more concerned about her right ankle. She was playing soccer over the weekend and heard a “pop.” She is able to bear weight, but it is uncomfortable. In determining the cause of the ankle pain, based on your knowledge of anatomy, what foot structures are likely involved? What other symptoms need to be explored? What are your differential diagnoses for ankle pain? What physical examination will you perform? What special maneuvers will you perform? Should you apply the Ottawa ankle rules to determine if you need additional testing?

Case 3: Knee Pain

Photo Credit: University of Virginia. (n.d.). Normal Knee Anatomy [Photograph]. Retrieved from http://www.med-ed.virginia.edu/courses/rad/ext/7knee/01anatomy.html. Used with permission of University of Virginia.

A 15-year-old male reports dull pain in both knees. Sometimes one or both knees click, and the patient describes a catching sensation under the patella. In determining the causes of the knee pain, what additional history do you need? What categories can you use to differentiate knee pain? What are your specific differential diagnoses for knee pain? What physical examination will you perform? What anatomic structures are you assessing as part of the physical examination? What special maneuvers will you perform?

With regard to the case study you were assigned:
  • Review this week’s Learning Resources, and consider the insights they provide about the case study.
  • Consider what history would be necessary to collect from the patient in the case study you were assigned.
  • Consider what physical exams and diagnostic tests would be appropriate to gather more information about the patient’s condition. How would the results be used to make a diagnosis?
  • Identify at least five possible conditions that may be considered in a differential diagnosis for the patient.

Note: Before you submit your initial post, replace the subject line (“Discussion – Week 8”) with “Review of Case Study ___.” Fill in the blank with the number of the case study you were assigned.

By Day 3 of Week 8

Post an episodic/focused note about the patient in the case study to which you were assigned using the episodic/focused note template provided in the Week 5 resources. Provide evidence from the literature to support diagnostic tests that would be appropriate for each case. List five different possible conditions for the patient’s differential diagnosis, and justify why you selected each. 

Note: For this Discussion, you are required to complete your initial post before you will be able to view and respond to your colleagues’ postings. Begin by clicking on the “Post to Discussion Question” link, and then select “Create Thread” to complete your initial post. Remember, once you click on Submit, you cannot delete or edit your own posts, and you cannot post anonymously. Please check your post carefully before clicking on Submit!

Read a selection of your colleagues’ responses.

By Day 6 of Week 8

Respond to at least two of your colleagues on 2 different days who were assigned different case studies than you. Analyze the possible conditions from your colleagues’ differential diagnoses. Determine which of the conditions you would reject and why. Identify the most likely condition, and justify your reasoning.

Submission and Grading Information

Grading Criteria

To access your rubric:

Week 8 Discussion Rubric

Post by Day 3 of Week 8 and Respond by Day 6 of Week 8

To Participate in this Discussion:

Week 8 Discussion


Assignment: Lab Assignment (Optional): Practice Assessment: Musculoskeletal Examination

A description of symptoms alone is not enough to form an accurate diagnosis of musculoskeletal conditions. Before forming a diagnosis, advanced practice nurses need to perform a physical examination. Although the musculoskeletal examination is relatively simple, it still needs to be performed multiple times before it can be mastered.

In preparation for the Comprehensive (Head-to-Toe) Physical Assessment due in Week 9, it is recommended that you practice performing a musculoskeletal examination this week.

Note: This is a practice physical assessment.   

To Prepare

  • Arrange an appropriate time and setting with your volunteer “patient” to perform a musculoskeletal examination.
  • Download and review the Musculoskeletal Checklist provided in this week’s Learning Resources as well as review the Seidel’s Guide to Physical Examination online media.

The Lab Assignment

Complete the following in Shadow Health:

  • Musculoskeletal (Practice)

What’s Coming Up in Week 9?

Photo Credit: [BrianAJackson]/[iStock / Getty Images Plus]/Getty Images

Next week, you will examine appropriate methods for assessing the cognition and the neurologic systems during your Discussion. You also will complete the last assessment, Comprehensive (Head-to-Toe) Physical Assessment. Once again, you will conduct this assessment in the Digital Clinical Experience using the simulation tool, Shadow Health. Make sure to plan your time accordingly.

Week 9 Required Media

Photo Credit: [fergregory]/[iStock / Getty Images Plus]/Getty Images

Next week, you will need to view several videos and animations in the Seidel’s Guide to Physical Examination as well as other media, as required, prior to completing your Discussion. There are several videos of various lengths. Please plan ahead to ensure you have time to view these media programs to complete your Discussion on time. 

Next Week

To go to the next week:

Week 9

S.

CC (chief complaint): “Lower back pain.”

HPI: D.T. is a 42-year-old Caucasian male with a chief complaint of lower back pain. He reports that the lower back pain began a month ago. He describes the pain as ‘stabbing’ and often radiates to his left leg. He reports that the pain sometimes causes numbness and weakness in the left leg. The low back pain is constant but worsens with activity, prolonged sitting, and bending. The patient has used OTC analgesic creams and Tylenol, which relieve the pain to some degree but recurs after some hours. He is concerned that the back pain may be long-term since it has already lasted a month. This will significantly interfere with his daily work activities. He rates the pain at 5/10.

Current Medications: OTC Diclofenac cream, applies twice daily. OTC Tylenol 500 mg TDS.

Allergies: Allergic to nuts- causes skin itching, redness, and swelling. No drug allergies.

PMHx: The patient has no history of chronic illnesses or admission. His immunization is not up to date. The last Flu shot was more than three years ago. Last TT- July 2018. He has received both Pfizer COVID-19 shots.

Soc Hx: D.T. is a lab technologist with a degree in Analytical chemistry. He is married and lives with his wife and two children, 15 and 10 years old. His hobbies include playing baseball and fishing. He is the captain of the baseball team in his organization. He denies smoking tobacco but reports taking a few whiskey glasses on weekends to wind up. He also denies any past or current substance use. The patient states that he is generally physically fit since he attends baseball practice 2-3 times a week. In addition, he eats balanced meals with a high composition of proteins and vegetables. He sleeps 5-6 hours a day. D.T. has private health insurance cover that also covers his family and is provided by his employer.

Fam Hx: The patient’s paternal grandfather died from prostate cancer at 82 years. His paternal grandmother died from an RTA at 85 years. His father has a history of high blood pressure. His younger brother has chronic asthma. The patient’s children have no chronic illnesses.

ROS:

GENERAL:  He denies fever, weight changes, or generalized body weakness.

HEENT:  Eyes:  Denies eye pain, excessive tearing, or blurred vision. Ears: Denies changes in hearing or ear pain. Nose: Denies nasal discharge, sneezing, or nose bleeds. Throat: Denies throat pain or hoarseness.  

SKIN:  Negative for skin rash, discoloration, or bruises.

CARDIOVASCULAR:  Negative for palpitations, chest tightness, swelling of lower limbs, or SOB on activity.

RESPIRATORY:  He denies difficulties in breathing, cough, wheezing, or sputum.

GASTROINTESTINAL:  He denies abdominal distress or changes in bowels.

GENITOURINARY:  Denies urinary symptoms of penile discharge. 

NEUROLOGICAL: Reports numbness and weakness in the left leg. Denies dizziness or loss of consciousness.

MUSCULOSKELETAL:  Reports lower back pain and left leg pain. Limited ROM on the left leg because of pain.

HEMATOLOGIC:  He denies bruising or a history of anemia.

LYMPHATICS:  Denies swelling of lymph nodes.

PSYCHIATRIC:  Reports increased stress due to prolonged back pain.

ENDOCRINOLOGIC:  Denies excessive sweating, heat and cold intolerance, excessive thirst or hunger, or increased urination.

ALLERGIES:  Hives when he eats nuts.

O.

Physical exam:

Vital Signs: BP- 122/78; HR- 86; RR-16; Temp-98.4; SPO2- 100%; HT-5’5; WT-167; BMI-27.8

GENERAL: White male patient in his 40s. The patient is calm and in no distress. He is well-groomed and appropriately dressed in casual wear. He exhibits positive facial expressions and body language. His speech is clear and goal-directed, with normal volume and rate.

CARDIOVASCULAR: Regular heart rate and rhythm.S1 and S2 are heard on auscultation. No gallop sounds or murmurs were heard.

RESPIRATORY: Smooth and even respirations. Symmetrical chest wall expansion. Lungs clear bilaterally.

NEUROLOGICAL: CN II – XII are intact; DTRs 2+ on the right leg and 1+ on the left leg. Normal sensation in the right foot; Reduced sensation in the left foot. Muscle strength 5/5 (right lower limb) 3/5 (left lower limb)

MUSCULOSKELETAL: Torso and head are upright. Normal balance when walking and standing, and the arms swing freely at the side. Straight leg raising elicits pain that radiates down the left leg when the left leg is slowly raised above 60°. On raising the right leg, the patient reports pain radiating down the left leg to the foot. ROM 4/5 in the left leg. Back ROM elicited pain with lateral rotation, forward flexion, and spine hyperextension.

Diagnostic results: Spine X-ray: Abnormal spine curve.

A.

Differential Diagnoses

Sciatica: Sciatica is characterized by pain along the sciatic nerve caused by compression of lumbar nerve roots in the lower back. Clinical features of sciatica include unilateral leg pain greater than low back pain; Pain radiating to the foot or toes; Numbness and paresthesia in the same distribution; Straight leg raising test elicits more leg pain; Localized neurology limited to one nerve root (Stynes et al., 2018). This is the most likely diagnosis due to positive symptoms of low back pain radiating to the left leg, numbness on the left foot, straight leg testing producing pain on the left foot, and crossed straight leg raising eliciting pain on the left foot.

Herniated lumbosacral disc: This is characterized by low back pain, limited trunk flexion, and sensory abnormalities at the lumbosacral nerve root distribution (Yu et al., 2022). The patient has low back pain, back pain with forward flexion, and an abnormal spine curve making Herniated lumbosacral disc a differential diagnosis.

Spinal nerve root impingement: This occurs when a spinal nerve root is compressed or irritated. The compression often causes high discomfort, like loss of sensation and weakness. When nerve root impingement occurs, the parts of the body that lie along the nerve’s path are usually the most severely affected (Berry et al., 2019). Compression of a spinal nerve root may have caused low back pain, left leg pain, and weakness. Besides, the abnormal spine curve may have irritated a nerve root.

Lumbar Spondylolisthesis: Lumbar Spondylolisthesisis characterized by intermittent and localized low back pain triggered by flexing and extending the affected segment (Dunn, 2019). The patient has low back pain and back pain with lateral rotation, forward flexion, and spine hyperextension, which are consistent with Spondylolisthesis.

Lower Back Muscle Spasm: Spasm of the lower back muscle is believed to produce secondary low-back pain and tenderness (Urits et al., 2019). This can be the cause of the patient’s lower back pain.

P.  

This section is not required for the assignments in this course (NURS 6512) but will be required for future courses.

References

Berry, J. A., Elia, C., Saini, H. S., & Miulli, D. E. (2019). A Review of Lumbar Radiculopathy, Diagnosis, and Treatment. Cureus11(10), e5934. https://doi.org/10.7759/cureus.5934

Dunn, B. (2019). Lumbar spondylolysis and Spondylolisthesis. Journal of the American Academy of PAs32(12), 50–51. DOI: 10.1097/01.JAA.0000604892.88852.c6

Stynes, S., Konstantinou, K., Ogollah, R., Hay, E. M., & Dunn, K. M. (2018). Clinical diagnostic model for sciatica developed in primary care patients with low back-related leg pain. PloS one13(4), e0191852. https://doi.org/10.1371/journal.pone.019185

Urits, I., Burshtein, A., Sharma, M., Testa, L., Gold, P. A., Orhurhu, V., Viswanath, O., Jones, M. R., Sidransky, M. A., Spektor, B., & Kaye, A. D. (2019). Low Back Pain, a Comprehensive Review: Pathophysiology, Diagnosis, and Treatment. Current pain and headache reports23(3), 23. https://doi.org/10.1007/s11916-019-0757-1

Yu, P., Mao, F., Chen, J., Ma, X., Dai, Y., Liu, G., … & Liu, J. (2022). Characteristics and mechanisms of resorption in lumbar disc herniation. Arthritis Research & Therapy24(1), 1-18. https://doi.org/10.1186/s13075-022-02894-8

Discussion: Assessing Musculoskeletal Pain

Patient Information:

Initials: C.A, Age: 46 years, Sex: Female, Race: African American

S.

CC (chief complaint): Bilateral ankle pain

HPI: C. A is a 46 is African American aged 46 years old. She presents with complaints of pain in her bilateral ankle, especially on the right side. She claims to have heard a “pop” while playing soccer over the weekend. She can tolerate weight, but it is painful. Her major worry is her right ankle.

Location: Bilateral ankle

Onset: Abrupt

Character: Sharp but not radiating pain

Associated signs and symptoms: Ankle enlargement and inadequate right ankle movement.

Timing: The pain has lasted for two days. It is irregular with each episode taking about 15-20 minutes.

Exacerbating/ relieving factors: Pain is intensified by walking, tolerating weight, or sitting. Pain is momentarily eased by cold compresses.

Severity: Pain is worse on the right ankle at about 5/10 compared to the left ankle which is about 2/10.

Current Medications: None

Allergies: No identified drug and food allergies

Past Medical History: No history of chronic medical conditions, blood transfusion, or previous surgeries. Flu vaccine: December 2020

Social History: She is a professional teacher and the team’s soccer captain. She likes playing soccer on weekends. She does not take alcohol, smoke tobacco, or use illicit drugs.

Family History: Her parents are all alive. Her mother is diabetic but effectively managed by metformin. She is not living with her husband because they separated two years ago. She has a 20-year-old college-going daughter.

ROS:

GENERAL:  No fever, chills, night sweats, or changes in weight

HEENT:  Eyes:  Refutes loss of vision, blurred vision, or yellow sclera. Ears, Nose, and Throat: Refutes ear discharges, hearing loss, dysphagia, nasal congestion, or sore throat.

SKIN:  Reports no rash, itching, or skin discoloration.

CARDIOVASCULAR: No paroxysmal nocturnal dyspnea, palpitation, chest pain, or orthopnea.

RESPIRATORY:  No cough, shortness of breath, sputum, or difficulty in breathing. GASTROINTESTINAL: No alteration in changes in abdominal distention, or bowel routines.

GENITOURINARY:  Refutes hematuria, frequency, or dysuria. The last menstrual period was on 07/09 /2022.

NEUROLOGICAL:  Denies convulsion, headache, syncope, or alterations in the functions of bowel and bladder.

HEMATOLOGIC:  No anemia, bruising, or bleeding.

LYMPHATICS:   No record of splenectomy. No lymphadenopathy.

PSYCHIATRIC:  Refutes anxiety, depression, hallucinations, or delusions.

ENDOCRINOLOGIC:  No cold, polydipsia, polyuria, and heat intolerance.

ALLERGIES: No history of asthma, eczema, or hives.

O.

Physical exam

Vital Signs: P 78 RR 19 Temp 98.4 F, BP 123/74 mmHg, Weight 128 lbs., Height 5′ 5″

General: A middle-aged female adult of African American origin. She has a minor discomfort. She is oriented and alert.

Respiratory: Vesicular breath sounds in entire lung zones, a symmetric chest that budges with respiration. No crackles or wheezing.

Cardiovascular: No murmurs. PMI in the fifth intercostal space, normoactive precordium, midclavicular line. S1 and S2 detected. Ecchymosis measuring 2 cm by 2 cm was noted around the lateral malleolus. Tenderness of the lateral malleolus was observed, particularly above the anterior talofibular ligament. Restricted range of motion of the right ankle, especially on plantar flexion, inversion, and dorsiflexion. Bilateral skin intact. No noted erythema or edema on the left ankle. The usual range of motion was noted on the left ankle. Noted bilateral constructive dorsalis pedis. Bilateral intact sensation, No noted deformity, crepitus, or bony tenderness.

Neurological: GCS 15/15, oriented to person, place, and time. Cranial nerves are intact, sensation in every dermatome is intact, and typical bulk, typical tone, and reflexes in all joints. Regular functions of bladder and bowel.

Diagnostic results:

The prone anterior drawer test: This test evaluates the reliability of the ankle’s lateral ligamentous complex. The test is crucial for the patient’s case.

Talar tilt test: This test focuses on the calcaneofibular ligament. The patient suffered pain around the ligament area.

Eversion test: This test is conducted to assess the reliability of the deltoid ligament. It is negative in the patient’s case.

Imaging: Based on the Ottawa Ankle rules, conducting a series of X-rays is crucial is necessary where the pain is noted in the malleolar area alongside any of the following signs; tenderness above the posterior periphery of the distal 6 cm or medial malleolus’ tip, tenderness above the posterior periphery of the distal 6 cm or lateral malleolus’ tip, and incapacity to tolerate weight shortly following an injury (Murphy et al., 2020). The patient, C.A, met the Ottawa rules. As a result, a right lateral X-ray was conducted, which indicated swelling in the soft tissue. There is a need for an MRI of the ankle or more perspectives to effectively describe the ligaments involved.

A.

Differential Diagnoses

Lateral Ankle Sprain: This pain is a frequent injury associated with sports. It occurs majorly with the ankle inversion and entailsthe lateral ligamentous complex, which comprises the calcaneofibular and posterior talofibular ligament, and anterior talofibular ligament that are damaged in reducing order (Martin et al., 2021). Patients with this condition often have a hematoma, tenderness above the sprained ligament, inadequate range of motion, and soft tissue swelling. These characteristics are common with the patient in this case. The “pop” sound she reported is an indication of a clear ligament tear.  As such, lateral ankle sprain is the primary diagnosis in this patient.

Ankle Fracture: This condition characterizes one or more ankle joint bones including the tibia, talus, and fibular. It presents as a cute immediate pain, tenderness, incapacity to tolerate weight, limited movement, pain, skin abnormalities, and swelling (McKeown et al., 2020). It is not the major diagnosis since ankle fractures are normally high-energy injuries but the patient can tolerate the weight.

 Syndesmotic Ankle Injury: This condition is also calleda high ankle sprain. It characterizes an injury to a minimum of one of the ligaments that encompass the distal tibiofibular syndesmosis (Raheman et al., 2022). It is also attributed to injuries associated with sports with an abrupt twisting force. It leads to more proximal pain above the ankle.

Anterior Impingement: This condition connotes strapped structures down thetibiotalar joint’s anterior margin in terminal dorsiflexion (Chen et al., 2019). It often characterizes ankle pain and restricted movement. It is also linked to considerable abnormalities in the osseous and soft tissues.

Achilles Tendinitis: This condition characterizes Achilles tendon inflammation. It manifests with swelling, pain, and erythema at the point of tendon placement into the calcaneus. It also manifested in incapability to move and tightness (Lee & Lee, 2018). In the case at hand, the patient reported pain and tenderness in the ankle’s lateral area. However, in Achilles tendinitis, the pain should manifest in the posterior area of the ankle.

P.  

This section is needless in this course. However, it will be necessary for future courses.

References

Chen, L., Wang, X., Huang, J., Zhang, C., Wang, C., Geng, X., & Ma, X. (2019). Outcome comparison between functional ankle instability cases with and without anterior ankle impingement: a retrospective cohort study. The Journal of Foot and Ankle Surgery, 58(1), 52-56. https://doi.org/10.1053/j.jfas.2018.07.015

Lee, Y. K., & Lee, M. (2018). Treatment of infected Achilles tendinitis and overlying soft tissue defect using an anterolateral thigh free flap in an elderly patient: A case report. Medicine, 97(35). Doi: 10.1097/MD.0000000000011995

Martin, R. L., Davenport, T. E., Fraser, J. J., Sawdon-Bea, J., Carcia, C. R., Carroll, L. A., … & Carreira, D. (2021). Ankle Stability and Movement Coordination Impairments: Lateral Ankle Ligament Sprains Revision 2021: Clinical Practice Guidelines Linked to the International Classification of Functioning, Disability, and Health From the Academy of Orthopaedic Physical Therapy of the American Physical Therapy Association. Journal of Orthopaedic & Sports Physical Therapy, 51(4), CPG1-CPG80. https://www.jospt.org/doi/10.2519/jospt.2021.0302

McKeown, R., Kearney, R. S., Liew, Z. H., & Ellard, D. R. (2020). Patient experiences of an ankle fracture and the most important factors in their recovery: a qualitative interview study. BMJ open, 10(2), e033539. http://dx.doi.org/10.1136/bmjopen-2019-033539

Murphy, J., Weiner, D. A., Kotler, J., McCormick, B., Johnson, D., Wisbeck, J., & Milzman, D. (2020). Utility of Ottawa ankle rules in an aging population: evidence for addition of an age criterion. The Journal of Foot and Ankle Surgery, 59(2), 286-290. https://doi.org/10.1053/j.jfas.2019.04.017

Raheman, F. J., Rojoa, D. M., Hallet, C., Yaghmour, K. M., Jeyaparam, S., Ahluwalia, R. S., & Mangwani, J. (2022). Can weightbearing cone-beam CT reliably differentiate between stable and unstable syndesmotic ankle injuries? A systematic review and meta-analysis. Clinical Orthopaedics and Related Research®, 10-1097. Doi: 10.1097/CORR.0000000000002171

Case 1: Back Pain

Episodic/Focused SOAP Note Template

Patient Information:

TJ, 42-year-old, Male, AA

S.

CC (chief complaint): Lower back pain

HPI: TJ is a 42 yar-old African American male who presents to the office today with cc of lower back pain that radiates to his left leg. TJ states the back pain has persist for the past month, reports sometimes the pain is sudden when waking in the morning and has a slower onset during the day after sitting for too long. TJ describes the pain as a shooting/burning, sometimes numbing pain, with the feeling of pins and needles to his left foot. TJ states when the pain is bothersome, he takes Motrin 200mg 3 tablets which usually gives some relief, TJ also states he does stretch exercises and takes short slow walks to help alleviate the discomfort. TJ rates his pain 7/10 on most days, today at the office TJ rates his pain 4/10 since he took Motrin 2 hours prior to visiting the clinic today.

Current Medications:

Loratadine 10mg by mouth daily – seasonal allergies

Men’s-one-a-day Multivitamin, 1 tablet daily

Motrin 200mg 3 tablets as needed – back pain

Allergies:

Seasonal allergies (itchy eyes, runny nose, sneezing)

Bananas (stomach cramping, nausea)

PMHx:

Tdap 2/2019

Annual Influenza Vaccine 9/2023

Hospital visit for jelly fish sting while on vacation in Aruba in 2020, no past surgical procedures, no past major or childhood illnesses.



Soc Hx:

Works at a linen company driving the work truck, delivering linen products to major restaurants and hotels, often works after hours.

Hobbies are soccer on the weekend with his work buddies and gaming a few hours after work to “de-escalate from the workday”.

Denies tobacco smoking, vaping, or chewing tobacco, denies illicit drug use, social drinker sometimes on the weekend with friends, at least 4 times a month, has 2-3 alcohol beverages in an occasion when out with friends.

TJ states he has a healthy living environment with his spouse, lives in a safe, quiet neighborhood predominately homeowners, no complaints with his work life, drives safely, uses his seat belt, denies texting and driving.

Fam Hx:

Mother: HTN, Eczema

Father: DJD, Disk fusion surgery in 2017, HTN

Sister: Hysterectomy in 2020

Maternal grandmother: Type 2 diabetes, Arthritis

Maternal grandfather: HDL, CAD

Paternal grandmother: Obesity, died in 2014 from stomach cancer.

Paternal grandfather: smoker, HTN, CHF

Son: healthy, age 14

ROS:

GENERAL:  Denies fever, chills, headache

HEENT:  Eyes: denies blurred/double vision. Ears, Nose, Throat: denies hearing impairment or ear pain, occasional runny nose and sneezing, denies throat pain

SKIN:  denies itchy or rash to skin

CARDIOVASCULAR:  Denies chest pain, chest pressure or chest discomfort. No palpitations or edema.

RESPIRATORY:  Denies SOB, cough  

NEUROLOGICAL:  c/o numbness, pins, and needles in left foot, burning and shooting sensation in left leg

MUSCULOSKELETAL:  c/o lower back pain that sometimes radiates down left leg

PSYCHIATRIC:  denies feeling sad, mood changes or anxiety

O.

Physical exam:

General: Pt is alert and oriented x4, sitting on exam table with a slight slump, often shifting sitting position, answers question appropriately, skin warm and dry to touch no rash or lesions noted, clothing appropriate for weather, no odor, teeth intact

HEENT: head symmetric, sclera white, PERRLA equal, no ear drainage, tympanic membrane w/o bulging, pearly gray in appearance, no nasal drip, throat symmetric, no mass or swollen glands noted.

Lungs/Chest: Lung sounds CTA, chest wall symmetric w/o deformity

Heart/Peripheral Vascular: NSR, S1 and S2, no gallop, radial and dorsalis pedis pulses +2

Musculoskeletal: c/o back pain on palpation, 5/5 in upper and lower extremities

Lymphatics: no swelling

Psychiatric: maintains good eye contact

Diagnostic results:

MRI of spine (captures herniated disc that could be causing sciatic pain)

Plain films of the lumbosacral spine: Spinal x-rays may be less detailed than other techniques, however they can detect tumors and bone spurs that may be pressing on the sciatic nerve (envrad.com).

A.

Differential Diagnoses:

Primary Diagnosis: Sciatica

Sciatica is a debilitating condition that results from a sciatic nerve root pathology, affected patients experience pain and paresthesia’s in the sciatic nerve distribution or an associated lumbosacral nerve root (Davis et al., 2022). Compression or pinching of one or more nerves extending from the L4 nerve root to the S2 nerve roots can cause sciatic pain. Pain is noted worse with bending, twisting, the individual can also experience numbness and tingling in the buttock that radiates down the individual’s leg. Sciatica is not a life-threatening condition however the pain can be debilitating and cause missed work and affect the individuals’ normal activities of daily living. More severe symptoms can relate to bowel and bladder incontinence. Sciatica can be managed with NSAIDS by decreasing inflammation. The sciatica nerve is responsible for the direct motor function of the lower extremity, which is why often patients experience discomfort to the lower extremities. Occupations where individuals are subjected to various awkward positions predisposes the individual, such as in TJ case being a truck driver, pulling dollies, and heavy lifting.

Herniated lumbosacral disc: Disc degeneration is usually the culprit for a herniated disc, this can be due to genetics, aging, or constant heavy lifting. Lumbar disc herniation results from several changes in the intervertebral disc including reduced water retention in nucleus pulposus, increased type 1 collagen ratio in the nucleus pulposus and inner annulus fibrosus, destruction of collagen and extracellular material, and an upregulated activity of degrading systems such as matrix metalloproteinase expression, apoptosis, and inflammatory pathways (Qaraghli and Jesus, 2023). Local inflammation and pressure enforced on the herniated disk is what causes the localized back pain. Impaired nerve roots such as the L5-S1 when compresses with a herniated disc manifest in increased discomfort to the sacral and buttock region that radiates from the perineum to the foot. Neurological maneuvers to test for causation of lower back pain is testing for a positive Lasegue sign or Bragaad’s sign. A positive Lasegue sign is a straight leg test with the individual lying on his/her back, the patient raises their leg and if pain and paresthesia is felt at an angle lower than 45 degrees that radiates below the knee it’s positive. The Bragaad sign is positive if increases sensitivity is felt while the patient dorsiflexes his/her foot while the examiner raises the leg. X-ray of the lumbar spine is the initial diagnostic test for ruling out pathologies.

Degenerative Joint disease: Degenerative Joint Disease (DJD) is another name for osteoarthritis. Generally, the cartilage that covers the bones in the joints thins away, the cartilage is needed for the cushion to prevent bone on bone rubbing which results in increased discomfort. Joints that can be affected are joints in the hands, knees, hips, neck, and back. Symptoms can range from muscle weakness, joint stiffness, limited range of motion, and crackling sounds heard during joint movement. DJD typically occurs in middle-aged and older adults, other risk factors are constant stress put on joints such as lifting weights, heavy lifting related occupation, obesity, and family history. DJD of the spine can manifest in pain and weakness of the neck and lower back that may extend to the arms and legs which can improve with rest. Initial treatment is nonsurgical, the provider may implement anti-inflammatory medication to control discomfort and lifestyle changes. Eventually if treatments fail surgical intervention may be optional with procedures such as joint realigning, removal of damaged joint lining, total joint replacement, and fusing the ends of the joints to limit motion and relieve pain (dignityhealth.org).

Spondylolisthesis: Spondylolisthesis is when the spinal vertebra slips over one another causing the individual pain, it generally occurs from the result of osteoarthritis. Symptoms consist of localized lower back pain and when compressed nerves is involved symptoms can involve neuropathic pain that radiates down the individuals leg. Some patients have little to no symptoms and don’t require intervention. Spondylolisthesis is not a dangerous condition; symptomatic treatment is primarily recommended. Nonsurgical treatments can involve a simple back brace to corticosteroid injections in the spine, if severe spinal surgery can be an option involving spinal decompression or spinal fusion surgery.

Muscle Spasm: Muscle spasms or cramps is an involuntary contraction of the muscle that occurs suddenly, usually resolves quickly, and are often painful (medicinenet.com). Painful, involuntary muscle contractions result from hyperexcitation of the muscle due to depletion of energy and fluid from the muscle cells. Muscle spasms are seen often in body builders especially those who exercise in hot environments, but it also poses as an occupation issue depending on job task such as working in extreme heat and strenuous engagement on the large muscles resulting from lifting and pulling. Other causes of muscle spasms can be related to electrolyte imbalances, atherosclerosis r/t inadequate blood supply to the muscles due to narrowed vessels, systemic illnesses such as DM, kidney disease and anemia, etc. Muscle spasms can be a generalized phenomenon. Eliminating causative factors is key to preventing muscle spasms, treating the underlying cause in most cases.

References

Al Qaraghli MI, De Jesus O. (2023). Lumbar Disc Herniation. https://www.ncbi.nlm.nih.gov/books/NBK560878/

Links to an external site.

Benjamin Wedro. (2023), Muscle Spasms, Medicine Net. https://www.medicinenet.com/muscle_spasms/article.htm

Links to an external site.

Davis D, Maini K, Vasudevan A. (2023). Sciatica. https://www.ncbi.nlm.nih.gov/books/NBK507908/

Links to an external site.

Dignity Health. Degenerative Joint and Bone Disease. https://www.dignityhealth.org/conditions-and-treatments/orthopedics/common-joint-injuries-and-conditions/degenerative-joint-and-bone-disease/diagnosis-and-treatment

Links to an external site.

How is Sciatica diagnosed? Envision Radiology. https://www.envrad.com/how-is-sciatica-diagnosed/

Name: NURS_6512_Week_8_Discussion_Rubric

 ExcellentGoodFairPoor
Main PostingPoints Range: 45 (45%) – 50 (50%) “Answers all parts of the Discussion question(s) with reflective critical analysis and synthesis of knowledge gained from the course readings for the module and current credible sources. Supported by at least three current, credible sources. Written clearly and concisely with no grammatical or spelling errors and fully adheres to current APA manual writing rules and style.Points Range: 40 (40%) – 44 (44%) “Responds to the Discussion question(s) and is reflective with critical analysis and synthesis of knowledge gained from the course readings for the module. At least 75% of post has exceptional depth and breadth. Supported by at least three credible sources. Written clearly and concisely with one or no grammatical or spelling errors and fully adheres to current APA manual writing rules and style.Points Range: 35 (35%) – 39 (39%) “Responds to some of the Discussion question(s). One or two criteria are not addressed or are superficially addressed. Is somewhat lacking reflection and critical analysis and synthesis. Somewhat represents knowledge gained from the course readings for the module. Post is cited with two credible sources. Written somewhat concisely; may contain more than two spelling or grammatical errors. Contains some APA formatting errors.Points Range: 0 (0%) – 34 (34%) “Does not respond to the Discussion question(s) adequately. Lacks depth or superficially addresses criteria. Lacks reflection and critical analysis and synthesis. Does not represent knowledge gained from the course readings for the module. Contains only one or no credible sources. Not written clearly or concisely. Contains more than two spelling or grammatical errors. Does not adhere to current APA manual writing rules and style.
Main Post: TimelinessPoints Range: 10 (10%) – 10 (10%) Posts main post by Day 3.Points Range: 0 (0%) – 0 (0%) N/APoints Range: 0 (0%) – 0 (0%) N/APoints Range: 0 (0%) – 0 (0%) Does not post main post by Day 3.
First ResponsePoints Range: 17 (17%) – 18 (18%) “Response exhibits synthesis, critical thinking, and application to practice settings. Provides clear, concise opinions and ideas that are supported by at least two scholarly sources. Demonstrates synthesis and understanding of Learning Objectives. Communication is professional and respectful to colleagues. Responses to faculty questions are fully answered, if posed. Response is effectively written in standard, edited English.Points Range: 15 (15%) – 16 (16%) “Response exhibits critical thinking and application to practice settings. Communication is professional and respectful to colleagues. Responses to faculty questions are answered, if posed. Provides clear, concise opinions and ideas that are supported by two or more credible sources. Response is effectively written in standard, edited English.Points Range: 13 (13%) – 14 (14%) “Response is on topic and may have some depth. Responses posted in the Discussion may lack effective professional communication. Responses to faculty questions are somewhat answered, if posed. Response may lack clear, concise opinions and ideas, and a few or no credible sources are cited.Points Range: 0 (0%) – 12 (12%) “Response may not be on topic and lacks depth. Responses posted in the Discussion lack effective professional communication. Responses to faculty questions are missing. No credible sources are cited.
Second ResponsePoints Range: 16 (16%) – 17 (17%) “Response exhibits synthesis, critical thinking, and application to practice settings. Provides clear, concise opinions and ideas that are supported by at least two scholarly sources. Demonstrates synthesis and understanding of Learning Objectives. Communication is professional and respectful to colleagues. Responses to faculty questions are fully answered, if posed. Response is effectively written in standard, edited English.Points Range: 14 (14%) – 15 (15%) “Response exhibits critical thinking and application to practice settings. Communication is professional and respectful to colleagues. Responses to faculty questions are answered, if posed. Provides clear, concise opinions and ideas that are supported by two or more credible sources. Response is effectively written in standard, edited English.Points Range: 12 (12%) – 13 (13%) “Response is on topic and may have some depth. Responses posted in the Discussion may lack effective professional communication. Responses to faculty questions are somewhat answered, if posed. Response may lack clear, concise opinions and ideas, and a few or no credible sources are cited.Points Range: 0 (0%) – 11 (11%) “Response may not be on topic and lacks depth. Responses posted in the Discussion lack effective professional communication. Responses to faculty questions are missing. No credible sources are cited.
ParticipationPoints Range: 5 (5%) – 5 (5%) Meets requirements for participation by posting on three different days.Points Range: 0 (0%) – 0 (0%) N/APoints Range: 0 (0%) – 0 (0%) N/APoints Range: 0 (0%) – 0 (0%) Does not meet requirements for participation by posting on three different days.
Total Points: 100 

Name: NURS_6512_Week_8_Discussion_Rubric

Don’t wait until the last minute

Fill in your requirements and let our experts deliver your work asap.